Sie sind auf Seite 1von 43

2014 NBE ONELINER

1. In the latent period of HIV infection, patient


develops:
1. Flu-like symptoms & lymphadenopathy
2. Progressive increase in HIV-1 viral titres
3. Progressive fall in CD4 counts
4. Systemic immune deficiency
2. Which of the following statements is TRUE
regarding occupationally acquired HIV infection in
health care workers?
1. Risk is greatest during the earliest and latest stages of
the disease
2. Principal route is by skin perforation with a solid
needle containing HIV infected blood
3. Post exposure HIV-prophylaxis should be started after
confirming HIV status of the source
4. Post-exposure HIV-prophylaxis is with' zidovudine
(250 mg bd) for 3 month
3. What is of utmost importance in treatment of
wound abscesses?
1. Broad-spectrum antibiotics
2. Adequate surgical decompression and curettage of
abscess
3. Primary closure of wound
4. Use of absorbable suture
4. A 38 year old obese lady develops pain &
swelling in her right leg along with fever & chills.
On examination, there is calor, rubor, dolor over a
poorly localized area over her calf with no pus
pointing. WBC count is 18000/cumm. Blood culture
is negative. Diagnose her condition:
1. Calf abscess
2. Lymphangitis
3. Cellulitis
4. SSSI
5. All of the following are characteristics of a
MAJOR wound infection, EXCEPT:
1. Discharge of infected serous fluid
2. Secondary drainage procedure may be required
3. May be associated with SIRS
4. Planned discharge to home may be delayed
6. A 40 year old man is operated for an abdominal
surgery via a midline abdominal incision. When is
he most likely to develop a wound abscess postoperatively?
1. 2-3 days
2. 7-9 days
3. 10-14 days
4. 14-21 days
7. Use of all of the following significantly decreases
airborne infection in operation theatre, EXCEPT:
1. Laminar air flow
2. Minimizing the no. of individuals in the OT
3. Air conditioning
4. High-efficiency particulate filters

8. Which of the following antiseptic is inappropriate for skin preparation of the operative site?
1. Chlorhexidine
2. Povidone Iodine
3. Cetrimide
4. Hexachlorophane
9. Definition of systemic inflammatory response
syndrome (SIRS) includes all, EXCEPT:
1. Hyperthermia (> 38C) 2. Bradycardia (60/min)
3. Tachypnoea (> 20/min) 4. WBC count < 4000/min
10. Following regarding Tropical chronic
pancreatitis is false:
1. Affects alcoholic elderly from lower socioeconomic
region esp. South India
2. Associated with ingestion
of cassava
3. Imaging studies reveal nodular fibrotic pancreas
with dilated ducts filled with stones
4. Surgery reserved for intractable pain
11. Regarding typhoid all are true, EXCEPT:
1. Caused by gram Negative bacillus
2. Diagnosis is by leucopenia, stool and blood cultures
and positive Widal test
3. Perforation of ulcer in 1st week carries poor
prognosis than 2nd week
4. Surgical intervention done in case of complications
12. For in order of minimize surgical site in the
operation theatre all are true, EXCEPT:
1. First scrubs should be for 3 mins.
2. Bacterial count to be kept below 10 CFU/cmm
3. Temp should below 19-22C and humidity between
45-55%
4. Good prepping and maintaining distance of
unscrubbed staff of atleast 50 cms from sterile field
13. Thymus gland abscess is congenital syphilis
are called:
1. Politizer's abscess
2. Fouchier's abscess
3. Duboi's abscess
4. Mycotic abscess
14. Malignant pustule occurs in:
1. Melanoma
2. Anthrax
3. Carbuncle
4. Actinomycosis
15. Following are true of eryiseplas, EXCEPT:
1. Streptococcal infection
2. Margins are raised
3. Commonly seen in temperate region
4. None of the above
16. Commonest cause of acute LN-adenitis in India
is:
1. TB
2. Lymphoma
3. Staphylococcal skin infection 4. Bare foot walking

2014 NBE ONELINER


17. Tetanus is caused by:
1. CI. Tetani
3. CI. Edematiens

3. GH levels decreased
4. Glucose tolerance decreased
2. CI. Welchi
4. CI. Septicum

18. 65 Kg male with 60% burns in catabolism is


admitted. An individual of this state requires 40
Kcal/kg/day and 2 gm of protein/day/kg. This young
man is given solutions having 20% glucose and 25%
protein. If 3000 ml/day is given, then:
1. Patient gets insufficient protein
2. Inadequate carbohydrate intake
3. Both protein and carbohydrate are adequate
4. Too much protein is infused
19. Metabolic complications during parenteral
nutrition include all, EXCEPT:
1. Hyperglycemia
2. Hypoglycemia
3. Hyperchloremic metabolic acidosis
4. Hypokalemia
20. Best method of post operative pain relief is:
1. NSAID suppositories
2. Intramuscular morphine
3. Acupuncture
4. Epidural analgesia
21. Metabolic features of
1. Low plasma insulin levels
2. Decreased hepatic gluconeogenesis
3. Increased hepatic glycogenolysis
4. Increased lipolysis
22. In Evidence Based surgery all are true,' EXCEPT:
1. Aims to provide practice of surgery on a logical and
scientific basis
2. Emphasis on preparing systemic review and keeping
them up-to-date them
3. Includes randomized control trials and studies
4. Level V evidence or grade of recommendation D is the
best evidence
23. For consent to be valid, informed it requires all,
EXCEPT:
1. Patient must be competent, not coerced into and well
informed of pros and cons of procedures and other
modalities available
2. Can be taken by junior member of the surgeons team
3. Explained in easy, lucid, language understood by the
patient
4. Incase of childrens/ mentally II patients, parents /
guardians are allowed for consent of procedure
24. In a person who has fasted for 5 days all are seen,
EXCEPT:
1. Free fatty acid levels in plasma increased
2. Immune reactive insulin level deceased

25. During nutritional assessment of surgical


patients status of muscle protein is indicated by
which one of following parameters:
1. Serum albumin
2. Triceps skin fold thickness
3. Hb level
4. Mid arm circumference
26. The type of naevus which is most likely to
undergo malignant change:
1. Intradermal naevus
2. Compound naevus
3. Blue naevus
4. Junctional naevus
27. Regarding Neurofibromatosis, false is:
1. Un encapsulated tumors of schwann cells
2. Resected along with nerve fibres
3. Association with axillary freckling and Lisch noduls
4. Always benign tumor
28. Spontaneous regression of malignant tumors is
seen with:
1. Neuroblastoma
2. Retinoblastoma
3. Malignant melanoma
4. All of the above
29. About cong torticollis all of the true, EXCEPT:
1. Associated with Breech deliveries
2. Untreated cases may lead to
plagiocephaly
3. Associated pterigium coli/Klieppel feil
syndrome
4. Always required surgical correction
30. Ephilis is:
1. Type of freckle
3. Infection caused by T.palidum

2. Dental Anomaly
4. Pregnancy tumor

31. In malignant melanoma, the margin of excision


for tumor size 2cms is :
1. 1 cms
2. 0.5 cms
3. 2 cms
4. 5 cms
32. Where are you most likely to use a full thicknes
graft?
1. Scrotum
2. Back
3. Scalp
4. Face
33. The best reconstruction after a segmental
mandibulectomy in a patient of lower alveolus
malignancy:
1. Skin grafting
2. Pectorals major myocutaneous flap
3. Delto pectoral flap
4. Free fibula flap
34. A deltopectoral flap is a type of :

2014 NBE ONELINER


1. Free flap
3. Fasciocutaneous flap

2. Myocutaneous flap
4. Fasciomyocutaneous flap

35. The blood supply for a transverse Rectus


abdominis myocutaneous flap comes from:
1. Internal mammary artery
2. Superficial epigastric artery
3. Deep inferior epigastric artery
4. Superficial external pudendal artery
36. Ideal graft for leg injury with 10 x 10 cm. exposed
bone :
1. Amniotic memb graft
2. Pedicle graft
3. Full thickness graft 4. Split thickness skin graft
37. Following are true of cultured bilayer skin
equivalent, EXCEPT:
1. More closely mimics normal anatomy
2. easily handled, can be sutured or meshed
3. Does not need secondary procedure
4. Long shelf life
38. Regarding Hemangiomas following are true:
1. Salman patch disappears after age one
2. Portwinestain present through life
3. Salman patch seen on forehead or over occiput
4. All are correct
39. Criteria for Brainstem death include the
following, EXCEPT:
1. Absence of corneal reflexes
2. Absence of motor response
3. Absence of spontaneous respiration after
preventilation with 100% O2 for atieast 5 minutes, pt is
connected from ventilator for 10minutes with PO2 > 60
mm of Hg
4. Tests performed by two clinicians on two separate
occasions with atleast one of them a consultant from the
transplant team.
40. Optimal storage time in hrs for liver is :
1. < 24 hrs
2. < 12 hrs
3. < 10 hrs
4. < 3hrs
41. In creation of peritoneum, gas used in
Laparoscopy is:
1. Coz
2. N20
3. Helium gas
4. All of the above
42. The intra abdominal pressure during
laparoscopy should be set between:
1. 5-8 mm Hg
2. 10 25 mm Hg
3. 20 25 mm Hg
4. 30 35 Hg
43. Principal cause of death in renal transplant
patients is:

1. Uraemia
3. Malignancy

2. Rejection
4. Infection

44. Hyperacute graft rejection is caused by :


1. B - lymphocytes
2. Macrophages
3. Preformed Antibodies
4. T-Iymphocytes
45. MOA of Sirolimus is :
1. Blocks-IL-2 gene transcription
2. Blocks IL-2 receptor signal transduction
3. Depletion and Blockade of T-cells
4. Prevents Iymphocyte proliferation
46. In renal transplantation in a living donor kidney,
Renal artery is anastomosed to :
1. Ext. Iliac Artery
2. Renal artery
3. Int. Iliac artery
4. Aorta
47. NOTES is :
1. Non obliterative trans esophageal surgery
2. Natural orfice trans esophageal surgery
3. Natural orfice trans luminal endoscopic surgery
4. Neo occlusive trans arterial Endo surgery
48. Triangle of doom has following Boundaries,
EXCEPT:
1. Vas deferens
2. Testicular vessels
3. Iliac vessels
4. Reflected peritoneal fold
49. Length of the laparoscopic Hand instruments
used/regularly in adult surgeries is:
1. 18cms
2. 28 cms
3. 25 cms
4. 36 cms
50. Graft-versus-host disease has occurred with the
transplantation of which of the following?
1. Kidney
2. Lung
3. Heart
4. Bone marrow
51. The following are true of congenital
lymphoedema, EXCEPT:
1. Lower limbs are affected more often
2. Usually unilateral
3. Onset usually occurs before puberty
4. Episodes of lymphangitis worsens the edema
52. In the management of leg ulcers, which of the
following is not done?
1. Cleaning the ulcer under tap water
2. Treating the skin of leg with emulsifying ointment
3. Use of topical antibiotics during dressing
4. Use of topical steroids to treat allergic response
53. Mr. Menon is due for a 10 hr flight to London. He
has undergone knee replacement surgery one month
ago. How best can he prevent DVT from taking place:

2014 NBE ONELINER


1. Taking sleeping tablets
2. Avoid alcohol
3. Walk in the aisle occasionally
4. Low-molecular weight heparin administered before
the flight
54. The commonest lymphangiographic finding in a
patient with lymphoedema praecox is:
1. Congenital hyperplasia of lymphatics
2. Proximal obliteration
3. Distal obliteration
4. Dysfunctional lymphatics
55. All of the following statement regarding filariasis
are TRUE, EXCEPT:
1. Commonest cause of lymphoedema world-wide
2. Microfilariae enter the blood at night
3. Diethylcarbamazine destroys the parasites and
reverses the Iymphatic changes
4. Wucheria bancrofti is responsible for 90% cases
56. Most accurate diagnostic technique in
lymphoedema is:
1. Lymphangiography 2. Isotope Iymphoscintigraphy
4. MRI
3. CT scan
57. Decongestive lymphoedema therapy includes all,
EXCEPT:
1. Skin care
2. Manual lymphatic drainage
3. Multilayer lymphoedema bandaging
4. Diuretics
58. Commonest cause of chyluria is :
1. Tuberculosis
2. Filariasis
3. Ascariasis
4. Malignancy
59. Meige's disease is :
1. Congenital lymph oedema
2. Ovarian tumor with pleural effusion and ascitis
3. Lymphaedema precox
4. Lymphagio sarcoma in chronic Iymphoedematous
limb.
60. Alemtuzumab is a chemo therapeutic agent with
MOA as:
1. Antibody against CD20 Ag
2. Antibody against CD 52 Ag
3. Tyrosine kinase receptor inhibitors
4. Farnesyl transferase inhibitors
61. Severe unilateral lymph edema is:
1. > 20 % excess limb volume
2. > 40 % excess limb volume
3. > 30 % excess limb volume
4. > 50 % excess limb volume

62. All the following are good prognosis features of


Hodgkins disease, EXCEPT:
1. Hb>10gm
2. Abs. lymphocyte count < 600/111
3. WBC < 15000/ cmm
4. Age < 45 years
63. Most malignant form of NHL is:
1. Diffuse large cell
2. Small cell lymphocytic
3. Follicular lymphoma
4. Large cell follicular
64. Podoconiosis is :
1. Type of fungal infections of feet
2. Endemic elephantiasis
3. Type of occupational chest infections
4. Malignancy of the nail
65. Odema pitting on pressure and disappearing an
bed rest and elevations is:
1. Grade II
2. Grade III
3. Latent or subclinical
4. Grade I
66. A young child was brought by her mother with
swelling in the lower posterior half of neck which
became prominent on crying. On examination the
margins more not well defined and fluctuation was
positive. Compressibility was possible diagnosis in
this patient is:
1. Cystic Hygroma
2. Bronchial cyst
3. Solitary lymph cyst
4. Cold abscess
67. Which of the following is not a feature of severe
limb ischaemia?
1. Pain in calf on walking
2. Rest pain
3. Coldness, numbness and paraesthesia
4. Ulceration and gangrene
68. Salim had a cut throat injury following a brawl.
He developed air embolism. Which of the following is
incorrect regarding treatment of this condition?
1. Placement of patient in Trendelenburg position
2. Oxygen administration
3. Left side placement of patient
4. Aspiration of left ventricle
69. False about fat embolism is:
1. Fat is metabolic in origin
2. Patient becomes comatose with small pupils
3. Retinal changes take place late after onset of disease
4. Petechial hemorrhages often occur
70. In all of the following, sympathectomy is effective,
EXCEPT one:
1. Intermittent claudication 2. Hyperhydrosis

2014 NBE ONELINER


3. Raynaud's disease

4. Causalgia

71. An ABI of less than suggests arterial injury even


in the presence of palpable pulses:
1. 0.7
2. 0.5
3. 0.3
4. 0.9
72. Gold standard for diagnosis of aortic rupture is:
1. 20 ECHO
2. X-ray chest PA view
3. Aor togram 4. Multi slice CT scan with contrast
73. While doing BK amputation, most important
technical consideration is :
1. Stump should be short
2. Ant flap longer than post flap
3. Fipula transected above the tibial
4. Nerves ligated at the level of muscles

2. Within the duct in the floor of mouth


3. At the opening of the submandibular duct
4. None of the above
80. Organism not found in acute bacterial parotitis is:
1. Haemophilus influenza
2. Staphylococcus aureus
3. Streptococcus viridans
4. None of the above
81. Which of the following is not used as a method for
prevention of Frey's syndrome during
parotidectomy?
1. Sternomastoid muscle flap
2. Lateral thigh flap
3. Temporalis fascial flap
4. Insertion of artificial membranes between skin & the
parotid bed

74. . Regarding Aortic dissection following is false:


1. Presents as tearing intrascapsular pain
2. Diagnosed by echo or CT/MRI with contrast
3. Control of BP is must before any further Imaging or
intervention
4. Type A usually are best managed medically

82. Rajmohan aged 70 years had a lesion in the


region of level II in the neck which was cystic.
Incorrect statement regarding this condition is:
1. This is cystic degeneration from a small undetected
primary squamous carcinoma in tonsil or tongue base
2. Radiotherapy is required
3. Laser excision may be done
4. There is no need for neck dissection

75. Cimino fistula is a fistula created between:


1. Radial A and cephalic vein
2. Ulnar A and vein
3. Subclavian A and vein
4. Long Saphenous vein and femoral artery

83. New man and seabrock's Sx (surgery) is done for:


1. Cleft palate
2. Cleft lip
3. Repair of parotid fistula
4. Chronic Iymphoedema

76. Commonest site for ectopic salivary gland


tumor is:
1. Tongue
2. Cheek
3. Palate
4. Neck
77. One of the following is not a reliable indicator
of malignant change in submandibular salivary
gland:
1. Facial nerve weakness
2. Pain in submandibular region
3. Rapid enlargement of swelling
4. Cervical node enlargement
78. Warthin's tumor is :
1. Malignant neoplasm
2. Rapidly growing
3. Gives a hot pertechnetate scan
4. Cold pertechnetate scan
79. Carol presents with an acute painful swelling
in the region of the submandibular gland, which is
precipitated by eating & resolves spontaneously
over a period of 1-2 hours after meals. This would
mean presence of stone in:
1. The hilum of the gland

84. Ca of buccal mucosa commonly drain in to


following lymph nodes sites:
1. Submental
2. cervical
3. Submandibular
4. Supraclavicular
85. A patient of Ca of Rt lateral border of tongue with
level III lymph nodes on left side of a size 5 cms
staging is :
1. No
2. N1
3. N2
4. N3
86. Earliest tumor of appear after birth is :
1. Stemomastoid tumor
2. Ranula
3. Cystic hygroma
4. Thyroglossal cyst
87. True about carotid body tumour is :
1. Arises form pharyngeal wall
2. FNAC is done for diagnosis
3. May metastasize
4. Young adults affected
88. Muco epidermoid Ca of parotid arise from:
1. Secrertory cells
2. Myoepithelial cells
3. Excretory cells
4. Myofibrils

2014 NBE ONELINER


89. Which of the following statements regarding
chronic hyperplastic Candidiasis is not true?
1. Lesions are common at oral commissures
2. Nystatin & amphotericin eliminate the infection
3. Re-infection after treatment is a definite
problem
4. Surgery is never required
90. Which of the following lesions is associated with
an increased incidence of oral malignancy?
1. Oral Lichen planus
2. Oral submucous fibrosis
3. Dyskeratosis congenital
4. Chronic hyperplastic candidiasis
91. Regarding palatal cancers, all are true, EXCEPT:
1. Commonly seen where reverse smoking is
practiced
2. Most of the tumours are of minor salivary gland
origin
3. They present as sessile swellings &
ulcerate late
4. Deep infiltration into the underlying bone is
very common
92. Investigation of choice for diagnosis of
oropharyngeal cancer:
1. Computerized tomography 2. MRI
3. Radiography
4. Radionuclide studies
93. Reconstruction after low-level maxillectomy for
a-hard palate can be carried out by :
1. Latissimus dorsi flap
2. Vascularised iliac crest graft
3. Fibular flap
4. All of the above
94. Resection of mandible due to gross tumour
invasion is followed by primary reconstruction by
which of the following methods?
1. Using a vascularised bone
2. Free corticocancellous graft
3. Alloplastic system supplemented with
cancellous bone mush
4. All of the above
95. Invasion of the edentulous mandible in
carcinoma of the floor of the mouth is :
1. By deficiencies in the cortical bone of the alveolar crest
2. Via the periodontal ligament
3. Above the insertion of the mylohyoid muscle
4. All of the above
96. Which statement regarding nasopharyngeal
carcinoma is not true?
1. Commonest variety is adenocarcinoma

2. Commonly seen in the Cantonese


3. Epstein-Barr virus is the infective agent
4. Salted fish consumption is an important cause
97. True about glomus-jugulae tumor, EXCEPT:
1. M.C. in male
2. Arise from non-chromaffin cells
3. Fluctuating tinnitus and conductive type
deafness is earliest symptoms
4. CT scan salt and pepper appearance of involved bone
98. A patient with Ca tongue is found to have lower
neck positive nodes. The Rx of choice for the lymph
nodes is :
1. Radical neck dissection
2. Tele radiotherapy
3. Suprahyoid neck dissection
4. Lower cervical neck dissection
99. Which Ca has best prognosis:
1. Ca. lip
2. Ca check
3. CA tongue
4. Ca palate
100. Treatment of stage T3 N1 of Ca. maxilla is :
1. Radiation only
2. Chemo + radiation
3. Sx with radiation
4. Chemo Rx only
101. MEN II A includes:
1. Ganglioneuromas 2. Cutaneous Lichen Amyloids
3. Hypocalcemia
4. Mutation in chromosome 10
102. During bilateral adrenalectomy ,
intraoperative dose of hydrocortisone should be
given after
1. Opening the abdomen
2. Ligation of left adrenal vein
3. Ligation of right adrenal vein
4. Excision of both adrenal glands
103. Commonest cause of Cushing's syndrome is:
1. Bilateral adrenal hyperplasia
2. Adrenal adenoma
3. Adrenal carcinoma
4. Steroids
104. Most common cause of Addison's disease is :
1. Tuberculosis
2. Metastatic carcinoma
3. Autoimmune
4. Amyloidosis
105. Which of following is preoperative preparation
pheochromocytoma?
1. Fluids
2. Phenoxybenzarmine
3. Nifedepine
4. propanolol
106. Regarding Adrenal Incidentalomas following is
false:

2014 NBE ONELINER


1. Detected in 4% patients on Imaging studies and
prevalence increases with age.
2. Majority of them are phaechromocytomas
3. Hormonal evaluation is required
4. Smaller masses usually are followed up
107. Malignant phaeochromocytoma differ from
benign by ai, EXCEPT:
1. Breached capsule
2. Vascular Invasions and mets
3. Ki-67 positive cells in high numbers
4. Increased adrenaline levels
108. Following statements about
craniopharyionmias is true:
1. Tumors are uniforming solid
2. Usually malignant
3. May cause compressingof optic tracts and visual
symptoms
4. Children with these lesion develop acromegaly
109. Opsomyoclonus is encountered as elf of
1. Memingioma
2. Neuroblastoma
3. Von tripped lindou disease 4. Neurofibromato
110. Which of the following is the mc type of pituitary
yadenome?
1. Thyrotropinoma
2. Gonadotropinoma
3. Prolactinoma
4. Corticotropinoma
111. A blood stained discharge from the nipple
indicates:
1. Breast abscess
2. Fibroadenoma
3. Duct papilloma
4. Fat necrosis of breast
112. A Female Patient present with a hard , mobile
lump in her rt. Breast . Which investigation would be
most helpful in making the diagnosis ?
1. FNAC
2. Needle biopsy
3. Excision Biopsy
4. Mammography
113. On mammogram all of the following are the
features of a malignant tumor, EXCEPT:
1. Spiculation
2. Microcalcification
3. Macrocalcification 4. Irregular mass
114. Which of the following stage of Breast Ca
corresponds with following feature ~ Breast mass of
6 x 3 cm. size with hard mobile ipsilateral axillary
lymph node and ipsilateral supraclavicular lymph?
2. T3N1 M1
1. T4 N2 Mo
3. T4N1 M1
4. T3 N3 Mo
115. Increased incidence with prolonged breast
feeding Breast a which is multicentric and bilateral:
1. Ductal
2. Lobular

3. Mucoid

4. Colloid

116. All are TRUE about CA breast, EXCEPT:


1. Affected sibling is a risk factor
2. Paget's disease of nipple is Intraductal type of CA
3. Common in aged nulliparous
4. Increased incidence with prolonged breast feeding
117. Peau d'orange is due to :
1. Arterial obstruction
2. Blockage of subdermal lymphatics
3. Invasion of skin with malignant cells
4. Secondary infection
118. Malti, a 45 years female patient with a family hlo
breast carcinomas, showed diffuse microcalcification
on mammography. Intraductal carcinoma is situ was
seen on biopsy. Most appropriate management is :
1. Quadrantectomy
2. Radical mastectomy
3. Simple mastectomy
4. Chemotherapy
119. Breast conservation surgery is indicated is one
of the following conditions:
1. T1 breast tumor
2. multicentric tumor
3. Extensive in situ cancer
4. T4b breast tumor
120. Cystosarcoma phylloides is treated by :
1. Simple mastectomy
2. Radical mastectomy
3. Modified radical mastectomy
4. Antibiotic with conservative treatment
121. All of the following are used for reconstruction
of breast, EXCEPT :
1. Transverse rectus abdominis myocutaneous flap
2. Latissimus dorsi myocutaneous flap
3. Pectoralis major myocutaneous flap
4. Transverses rectus abdominis free flap
122. A 14 year old healthy girl of normal height and
weight for age, complains that her right breast has
developed twice the size of her left breast since the
onset of puberty at the age of 12. Both breasts have a
similar consistency on palpation with normal nipples
areolae. The most likely cause for these findings is :
1. Cystosarcoma phyllodes
2. Virginal hypertrophy
3. Fibrocystic disease
4. Early state of carcinoma
123. Large breast is not seen in :
1. Filariasis
2. Giant fibroadenoma
3. Cystosarcoma phylloides 4. Schirrhous carcinoma
124. For pregnant women who are found to have
breast cancer :
1. Carcinoma of the breastbehaves more aggressively in
pregnant women owing to hormones stimulation

2014 NBE ONELINER


2. Breast conservation is inappropriate for third
trimester pregnancies
3. Most will have hormonally sensitive tumors
4. Administration of adjuvant chemotherapy is safe for
the fetus during the second and third trimesters
125. True statements regarding Pagets disease of
the breast include that is :
1. Usually precedes development of pagets disease of
bone
2. Presents with nipple areolar eczematous changes
3. Does not involves axillary lymph nodes because it is a
manifestation of intraductual carcinoma only
4. Accounts for 10to 15 percent of all newely diagnosed
breast cancers
126. Which of the following electrolyte disturbances
are most common in the head injured patient?
1. Sodium
2. Potassium
3. Chloride
4. Bicarbonate
127. After a vehicular accident, Mohan went for a CT
scan. He was diagnosed to have a burst temporal
lobe. This would mean:
1. Chronic subdural haematoma
2. Contusional intracerebral haematoma
3. Extradural haematoma
4. None of the above
128. An 18 year old man is admitted to the
emergency room following a motorcycle accident. He
is alert and fully oriented but witnesses to the
accident report an interval of unresponsiveness
following the injury. Skull films disclose a fracture of
the left temporal bone. Following x-ray the patient
suddenly loses consciousness and dilatation of the
left pupil is noted. This patient should be considered
to have:
1. A ruptured berry aneurysm
2. Acute subdural hematoma
3. Epidural hematoma
4. Intraabdominal hemorrhage
129. The term post traumatic epilepsy refers to
seizures occurring:
1. Within moments of head injury
2. Within 7 days of head injury
3. Within several weeks to months after head injury
4. Many years after head injury
130. Which of the following is commonest source of
Extradural hemorrhage?
1. Middle meningeal artery
2. Subdural venous sinus
3. Charcot's artery
4. Middle cerebral artery

131. 58 years old male presented with symptoms &


Signs of progressive neurological deficits altered
sensorium and irritability. On inquiry there was a
h/o trauma 3 weeks back. Most likely diagnosis is:
1. Acute subdural haematoma
2. Chronic subdural haematoma
3. Extradural Haematoma
4. Brain tumor
132. In multiple metastatic tumors of the brain ,
which of the following techniques is the most
plausible option for treatment ?
1. Surgery
2. Stereotic radiosurgery
3. Radio therapy
4. Steriods
133. After a recent office visit for evaluation of
headaches accompanied by nausea and vomiting and
progressive truncal ataxia , a 4 year old boy is
diagnosed with medulloblastoma. Which of the
following statements regarding these tumors is true?
1. Medulloblastoma is one of the rarer primary brain
tumors in children
2. First line treatment should include surgical debulking
with radiation therapy
3. Chemotherapy with carmustine is effective at inducing
remission of medulloblastomas
4. Most of these tumors arise from the floor of the fourth
ventricle
134. A 45 year old woman with a long history of
headaches has a large olfactory groove mass on
computed tomography (CT) scan. (SELECT 1 TUMOR)
1. Ependymoma
2. Oligodendroglioma
3. Meningioma
4. Glioblastoma multiforme
135. A middle aged man presents with a long history
of seizures and a calcified frontal lobe lesion.
(SELECT 1 TUMOR) :
1. Ependymoma
2. Oligodendroglioma
3. Meningioma
4. Glioblastoma multiforme
136. A 25 year old woman presents with worsening
hearing loss and tinnitus. A computed tomography
(CT) scan reveals tumor involvement of cranial nerve
VIII bilaterally. What is it likely to be ?
1. Von Hippel-Landau disease
2. Prolactin-secreting adenoma
3. Neurofibromatosis type II
4. Growth hormone-secreting adenoma
137. In raised ICP with disrupted blood-brain barrier
, which of the following drugs should be used with
caution?
1. Steroids
2. Mannitol
3. Barbiturates
4. Frusemide

2014 NBE ONELINER


4. Urine / serum creatinine
138. A 6 Year old boy is found to have pigmented
hamartomatous lesions of the iris and an optic
glioma upon fundoscopic examination. Your
diagnosis is :
1. Von Hippel-Landau disease
2. Prolactin-secreting adenoma
3. Neurofibromatosis type I
4. Growth hormone-secreting adenoma
139. A 25 year old woman presents with complaints
of significant weight gain, amenorrhea, and "purple
lines" on her skin. Select one of the following:
1. Von Hippel-Landau disease
2. Prolactin-secreting adenoma
3. Neurofibromatosis type II
4. Growth hormone-secreting adenoma
140. Suprasellar calcification with polyuria seen in:
1. Langerhan cell histocytosis
2. Medulloblastoma
3. Pinealoma
4. Craniopharyngioma
141. Not true regarding Dandy Walker cyst:
1. Cerebellar vermis Hypoplasia
2. Posterior fossa cyst
3. Long tract signs (+)
4. Arachnoid cyst
142. True about Berry Aneurysm through is
following, EXCEPT:
1. Mc site of rupture is apex which cause SAH
2. Wall contains
smooth
muscle fibroblasts
3. 90% occurs in Ant port of circulation at branching
points
4. Occasionally associated with NF1, coarctation of aorta.

146. Regarding Hypothermia, following is true,


EXCEPT:
1. Risk of
ventricular
fibrillation increases at
temp < 28 C.
2. Cardiac arrest occurs at 20C
3. Diagnosis confirmed by Rectal temp of 35 C
4. At temp < 30C, heart is responsive to
147. defibrillation and inotropes
Half life of platelets is :
1. 24 hrs
2. 48 hrs
3. 5 days
4. 9 days
148. Mc symptom of hemolytic transfusion a
conscious patient is :
1. Pyrexia and rigors
2. Hemoglobinuria and oliguria 3. Burning sensation and
pain in
3. the limb
4. Urticaria and pruritis
149. Following is the best parameter in the
management of shock:
1. Blood Pressure
2. Pulse oximetry
3. Deficiency of effective of circulation
4. CVP
150. Manohar admitted for leiomyoma of the
stomach was to undergo elective surgery. However,
he was diagnosed with Christmas disease. He will
require which of the following blood fractions?
1. Platelet concentrate
2. Fresh frozen plasma
3. Cryoprecipitate
4. None of t

143. Witzelschult syndrome is seen in :


1. Temporal lobe tumor
2. Parietal lobe tumor
3. Frontal lobe tumor
4. Head injury

151. Ormond's disease is caused by all, EXCEPT:


1. Hodgkin's disease
2. Inflammatory bowel disease
3. Ca breast
4. Focal glomerulosclerosis

144. Secondary haemorrhage differs from reactionary haemorrhage in that:


1. It is due to rolling of a ligature
2. It follows restlessness, coughing & vomiting which
raise the venous pressure
3. It occurs due to infection & sloughing of part of the wail
of an artery
4. It takes place within 24 hours

152. Which of the following is not an anatomical site


of narrowing where a ureteric stone is arrested?
1. Crossing the vas deferens
2. Uretero pelvic junction
3. Crossing the iliac artery
4. Before entering the bladder wall

145. Characteristic finding of perenal azotemia in a


post operative patient is :
1. Urine sodium of 28 meq/L
2. Urine chloride of 15 meq/L
3. Fractional excretion of sodium less than1

153. Strangury is due to :


1. Mid ureter
2. Urethra
3. Inflammed bladder 4. Stretching of renal capsule
154. True regarding ureteric stones is all,
EXCEPT?

2014 NBE ONELINER


1. Urine is always infected
2. Pain is referred to tip of penis in
intramural stones
3. Source is always the kidneys
4. Expectant treatment may be useful
155. The following statements about the YAG laser is
correct, EXCEPT?
1. It can even cut the wire of stone baskets
2. Its use for uric acid stones has caused deaths due to
generation of cyanide
3. It has a wavelength of 2100 nm
4. It is effective against the hardest urinary stones
156. Tumor staging and grading in early bladder
cancer is usually done with following
1. USG
2. CT Scan
3. Cystography
4. Transurethral resection
157. A 10-mm calculus in the right lower ureter
associated with proximal hydroureteronephrosis in
45 year old best treated with :
1. Open ureterolithotmy
2. Extracorporeal shockwave lithotripsy
3. Antegrade percutaneous access
4. Ureteroscopic retrieval
158. Squamous cell tumor of urinary bladder is most
consistently associated with:
1. Stone
2. Schistosomiasis
3. Chr. Cystitis
4. Diabetes mellitus
159. True about bladder stones is all, EXCEPT:
1. Rare in Indian children
2. Primary stones are rare
3. Small stones can be removed per urethra
4. Maximum stones are radioopaque
160. False about Balkan nephropathy is :
1. Squamous cell carcinoma
2. Involves upper urinary tract
3. Associated with consumption of grains stored in damp
environment
4. Nephron sparing surgery is done over nephrectomy
161. Tear-drop bladder is a feature of :
1. Tuberculosis
2. Hunner's ulcer
3. Perivescial hemorrhage with rupture
4. Perivesical hemorrhage without rupture
162. One of the following disease will show urinary
bladder calcification radiologically which resemble
fetal head in pelvis:
1. Tuberculosis
2. Schistosomiasis

3. Chronic cystitis
4. Malignancy
163. To differentiate between stress
incontinence and Detrusor instability
investigation done is :
1. Cystourethroscopy
2. Urodynamic study
3. MCU
4. Retrograde urethroscopy
164. For treatment of the ectopia vesicae,
which of the following bone is divided to
reach the site :
1. Pubic ramic
2. IIiac bone
3. Ischium bone
4. Symphysis
165. A patient Kailash presents with haematuria for
many days. On investigations he is found to have
renal calculi, calcifications in the wall of urinary
bladder and small contracted bladder. Most probable
cause is :
1. Schistosomiasis
2. Amyloidosis
3. Tuberculosis
4. Ca urinary bladder
166. About ectopic vesicae, following is true, EXCEPT:
1. Ca bladder may occur
2. Ventral curvature of penis
3. Incontinence of urine
4. Visible uretero - vesicle efflux
167. Rahul developed a neuropathic bladder after
an accident. The lesion is found above T10. He has
incomplete bladder emptying but a good capacity
bladder. He can be managed by :
1. Condom drainage
2. Clean intermittent
3. catheterisation
4. Endoscopic sphincterotomy
5. Bladder reconstruction with fitment of artificial
urinary sphincters
168. Most common complication of ileal conduit as
a method of permanent urinary diversion is:
1. Ureteroileal stricture
2. Stenosis at the percutaneous site
3. Urine infection
4. Reabsorption of urine
169. Komal complained of urine loss during
laughing & sneezing. Which of the following is
FALSE regarding her condition?
1. It is usually seen in those with H/O caesarian
section
2. History of difficult labour with use of forceps
3. Seen with epispadias

2014 NBE ONELINER


4. Symptoms may change with menstrual cycle
170. Which of the following drugs is used for non-adjuvant chemotherapy for bladder causes?
1. BCG
2. Gemcitabine
3. Doxorubicin
4. Vinblastine
171. Diagnosis of bladder diverticulum is best
made on :
1. Cystoscopy with partially distended Bladder
2. Intravenous urography
3. Retrograde cystography
4. Cystoscopy with fully distended Bladder
172. Following are the most common sites of
Ca bladder:
1. Lateral walls
2. Trigone
3. Both of the above
4. None of the above
173. The following are known causes of UTI in
females, EXCEPT:
1. Urethral stricture
2. Colonization of perineal skin by E.Coli
3. Neurogenic bladder dysfunction
4. Pre menopausal status
174. Carcinoma in schistosomiasis of the
bladder commences from which of the
following bilharzial lesion ?
1. Papilloma
2. Nodule
3. Ulcer
4. Pseudo-tubercle
175. characteristics of the urethral syndrome
does not include:
1. Symptoms of UTI
2. Negative urine culture
3. Absent pus cells in urine
4. Interstitial cystitis
176. True phimosis consist of following, EXCEPT:
1. Fissuring of the preputial skin
2. Balanitis xerotica obliterans
3. Scarring of the prepuce
4. Physiological adhesions
177. Excision of fibrous plaque in Peyronie's disease
cannot be replaced by:
1. Dermal graft
2. Vein graft
3. Tunica vaginalis graft
4. Non absorbable sutures
178. The condition which is a carcinoma-in-situ is all,
EXCEPT:
1. Pagets disease of the penis
2. Giant condylomata accuminata

3. Erythroplasia of Queyrat
4. Bowen disease
179. Rehman 33 year male presented with carcinoma
of penis. At presentation there were no nodes
palpable in the groin. However he developed nodes
in the right. side of groin 2 months after surgery.
Further treatment should include:
1. Observation
2. Antibiotics
3. Chemotherapy & radiation
4. Unilateral ilioinguinal node dissection
180. Malignant transformation occurs in ulcer after
many years of which of the following STI in the penis?
1. Lymphogranuloma venereum
2. Granuloma inguinale
3. Condylomata acuminata
4. None of the above
181. Androgen production from a cryptorchid testis
at 16 years of age is :
1. Reduced to half of normal output
2. Not reduced at all
3. Reduced by 30%
4. Reduced by 75%
182. Typical of Buschke-Lowenstein tumor is all,
EXCEPT:
1. Treatment is surgical
2. Locally destructive;
3. Spreads to lymph nodes>
4. It is a verrucous carcinoma
183. The most common cause of priapsm in of the in
recent years is :
1. Leukemic infiltration of pains
2. Spinal cord trauma
3. Sickle cell disease
4. Intracavernous injectiontheraphy
184. Balanoposthitis is associated with all of the
following, EXCEPT:
1. Penile cancer
2. Psoriasis
3. Gonorrhoea
4. Lichen planus
185. Chemotherapeutic drug used against penile
calJper are all, EXCEPT:
1. Etoposide
2. Cisplatin
3. Methotrexate
4. Bleomycin
186. Non-filarial elephantiasis of the scrotum is due
to infection with:
1. HIV
2. Lymphogranuloma venereum
3. Chancroid 4. Syphillis

2014 NBE ONELINER


187. The following statement is false about
peyronie's disease:
1. Patients present with complaints of painful erection
2. The condition affects adolescent males
3. The condition can be associated with Dupuytren's
contracture of the tendons of the hand
4. Spontaneous regression occurs in 50% of the cases
188. A patient comes with stage III nonserninomatous testicular tumor treatment of choice
is:
1. Radiotherapy
2. Chemotherapy
3. Hormonal therapy
4. Surgery
189. A 12 year old boy Naman presents with acute
onset right scrotal pain. The pain is not relieved on
elevation of the scrotum. The testis is enlarged and
tender. There is no history of trauma. Which of the
following is the most appropriate management?
1. Immediate exploration
2. Antibiotics
3. Psychiatric evaluation
4. Antibiotics and scrotal elevation
190. A patient .presented with a hard swelling in his
right testis. All are true statements, EXCEPT:
1. Trans scrotal biopsy is needed
2. Inguinal exploration is done
3. High inguinal exploration should be done
4. Scrotal U/S is done
191. Not true about carcinoma penis is :
1. Erythroplasia of Queret is a precancerous condition
2. 40% of patients are under 40 years of age
3. Circumcision if done anytime before puberty provides
100% protection against carcinoma penis
4. More than 50% patient have inguinal enlargement
when they present
192. The most common non-germ cell tumor of the
testis is :
1. Leydig cell tumor 2. Gonadoblastoma
3. Sertoli cell tumor 4. None of the above
193. Besides LDH which of the following tumor
marks are elevated in seminoma?
1. AFP
2. hCG
3. Both of the above 4. None of the above
194. The most common bilateral tumor of the testis
is:
1. Teratoma differentiated
2. Seminoma
3. Yolk sac tumor
4. Malignant lymphoma

195. Patients should be advised to continue their


normal contraceptive precautions after vasectomy
for a period of :
2. 8-10 weeks
1. 4-8 weeks
3. 12-16 weeks
4. 16-20 weeks
196. A 32 year male was having tender pain in his
right scrotum ; He has been on AKT for pulmonary
Kochs since 1 month . All are false regarding his
clinical picture , EXCEPT :
1. There is a lax secondary hydrocele in
80% of cases
3. Beading of vas is a characteristic feature
4. Seminal Vesicle is normal
5. A cold abscess could form in early cases
197. Treatment of carcinoma-in-situ of testicular
cancer in 40/M diagnosed by biopsy is :
1. Surgery
2. External beam radiation
3. Chemotherapy
4. Interstitial radiation
198. Vithal 301M was diagnosed as a clo high stage
seminoma after investigations & high inguinal
orchiectomy. One of the following drugs is not useful
for chemotherapy post surgery :
1. Vinblastine
2. Cisplatin
3. Dactinomycin
4. Taxol
199. Most common site of primary malignancy to
metastasise to the testis is :
1. Melanoma
2. Prostate
3. Kidney
4. Lung
200. Vishals semen analysis report showed an
ejaculate volume of 1.5ml & sperm Concentration of
21 million / ml. He had Suffered from mumps in
childhood :
1. Oligospermia
2. Normospermia
3. Azoospermia
4. None of the above
201. Rajnikant, 60 year old male is a case of Castomach. CECT Abdomen showed a mass measuring 4
cm x 4 cm in antrum with involvement of celiac
nodes. The management would be:
1. Palliative radiotherapy
2. Palliative chemotherapy
3. Radical subtotal gastrectomy
4. Total gastrectomy
202. A 17 year old boy underwent splenectomy and
cholecystectomy for hemolytic anemia. On third
post of day he developed sudden onset vomiting
and abdominal distension. The management
includes all of the following, EXCEPT :
1. Ryles tube insertion
2. Normal saline intravenously

2014 NBE ONELINER


3. Nil per oral
4. Surgery to relieve the distention
203. The true statement about gastric volvulus is :
1. Mesentero axial is commonest variety
2. Associated with Bochdaleck hernia
3. Endoscopy is investigation of choice for diagnosis
4. Transverse colon hemiates into thoracic cavity
204. Nilima, 50 year old female had an epigastric
lump and gastric outlet obstruction. Investigation
revealed lump arising from stomach and infiltrating
liver and pancreas with paraaortic lymph node
enlargement. UGIE Biopsy revealed poorly
differentiated. Adenocarcinoma. The best
management would be :
1. Palliative RT
2. Palliative CT
3. Radical total gastrectomy
4. Palliative gastrojejunostomy
205. Most common benign tumour of stomach is :
1. Leiomyoma
2. Adenomatous polyp
3. Epithelial polyp
4. Lymphoma
206. All of the following are indications for surgery in
gastric lymphoma, EXCEPT:
1. Bleeding
2. Perforation
3. Residual disease following chemotherapy
4. Intractable pain
207. Most common site for carcinoma stomach is :
1. Fundus
2. Body
3. Antrum
4. Lesser curvature
208. All the following are incorrect about CHPS,
EXCEPT?
1. Heller's myotomy is the required surgery
2. Bilious vomiting
3. Commoner in first female child
4. Hypochloremic alkalosis
209. Early dumping syndrome is characterized by
all of the following, EXCEPT:
1. Occurs in second hour after meal
2. Is aggravated by more food
3. Patient has raised hematocrit
4. Seen more commonly with Bilroth II
210. Mr. Sujit a 45 year old office executive, a known
case of Acid peptic disease presents with massive
upper G.I bleed. Endoscopy revealed a bleeding
duodenal ulcer. The best line of management would
be :
1. Embolisation

2. Cauterization of ulcers
3. Truncal vagotomy and pyloroplasty
4. Ligation of gastroduodenal artery
211. Commonest cause of duodenal fistula is :
1. As a complication of gastrectomy
2. An abscess connected with perforated duodenal ulcer
3. Traumatic rupture of duodenum
4. As a complication of Right colectomy
212. Which of the following should be avoided in
acute upper gastrointestinal bleed ?
1. Intravenous vasopressin
2. Intravenous blockers
3. Endoscopic Sclerotheraphy
4. Ballon Tamponade
213. In post gastrectomy nutritional syndrome
following occur , EXCEPT :
1. Calcium deficiency
2. Steatorrhoea
3. Constipation
4. vit. B12 defficiency
214. Most likely cause of loss of periodicity of
symptoms and sense of epigastric bloating in a case
of duodenal ulcer is :
1. Gastric outlet obstruction 2. Perforation
3. Carcinoma
4. Pancreatitis
215. In a case of hypertrophic pyloric stenosis, the
metabolic disturbance is :
1. Respiratory alkalosis
2. Metabolic acidosis
3. Metabolic alkalosis with paradoxical aciduria
4. Metabolic alkalosis with alkaline urine
216. A 25 year old office executive presents with
recurrent duodenal ulcer of 2.5 cm size. The
procedure of choice would be :
1. Truncal vagotomy
2. Truncal vagotomy with antrectomy
3. Highly selective vagotomy
4. Laparoscopic vagotomy and gastrojejunostomy
217. Treatment of choice for Duodenal atresia is :
1. Gastroduodenostomy
2. Duodenoduodenostomy
3. Duodenojejunostomy
4. Gastrojejunostomy
218. Correct statement about operation for morbid
obesity is:
1. Bacterial overgrowth in the By passed segment is a
complication of jejunoileal bypass
2. Following gastric bypass the patient may be
permitted to resume normal eating habits
3. Long term weight loss is not sustained after a
jejunoileal bypass

2014 NBE ONELINER


4. Ulceration in bypassed antrum has been a problem
after Roux-en-y gastric bypass operation.
219. Incorrect about primary duodenal diverticuli is :
1. Occur on concave border
2. Occur in 1st part of duodenum
3. Are usually single
4. Are accidental finding
220. Incorrect regarding mesentericoarterial
syndrome is:
1. Caused by compression of distended duodenum
2. Common in young females
3. Does not occur in obese patients
4. Most common in 5th - 7th decades
221. A post operative patient presents with duodenal
leak along with signs and symptoms of peritonitis.
The most appropriate management is :
1. Laparotomy with tube duodenostomy
and feeding jejunostomy
2. Peritoneal lavage with drains
3. Laparotomy and reanastomosis
4. Jejunostomy tube, duodenostomy tube and parenteral
nutrition.
222. Ladd's Band courses from:
1. Splenic flexure to ileocecal junction
2. 2nd part duodenum to right paracolic gutter
3. Caecum to sigmoid colon
4. OJ flexure to right sacroiliac joint
223. Following blunt abdominal trauma, a 12 year
old girl presents with upper abdominal pain, nausea
and vomiting, An upper gastro intestinal series
reveals a total obstruction of the duodenum with
coiled spring appearance in D2 - D3. The final
definitive management is :
1. Gastrojejunostomy
2. NGT suction and observation
3. Feeding jejunostomy
4. Evacuation of hematoma
224. Wind sock deformity is characteristic of :
1. Duodenal atresia
2. Rectal atresia
3. Biliary atresia
4. Oesophageal atresia
225. Most common type of Oesophageal perforation
is :
1. Traumatic perforation
2. Latrogenic perforation
3. Boerhaves syndrome
4. Corrosive injury
226. All are false regarding angiodysplasia of
colon, EXCEPT:
1. Usually found is 30-40 age group
2. Rectum is the commonest site

3. Selective angiography is diagnostic


4. Associated with aortic regurgitation
227. Abdominal actinomycosis usually manifests
after which surgery?
1. Hernia
2. Nephrectomy
3. Cholecystectomy 4. Appendicectomy
228. Most common complication of hyphoid is
1. Bleeding
2. Perforation
4. Paralytic ileus
3. Obstruction
229. The Waldeyer's fascia separates the rectum
from the:
1. Uterus
2. Lateral pelvic wall
3. Prostate
4. Sacrum
230. The chemotherapeutic agent of choice for
adjuvant treatment in rectal cancer in 60 year male
is :
1. 5 fluorouracil
2. Adriamycin
3. Paclitaxel
4. Epirubicin
231. Treatment of choice in colonic pseudoobstruction since 2 days in 40/m patient is :
1. Masterly inactivity
2. Colonoscopic decompression
3. Emergency colostomy
4. Emergency colectomy
232. Incidence of synchronous case of Ca colon is
malignancy in :
2. 15%
1. 5%
3. 25%
4. 50%
233. All are true regarding a caecal Volvulus, EXCEPT
1. Barium enema is usually diagnostic
2. Usually anticlock wise twist
3. Caecostomy may be required
4. May occur as part of volvulus Eonatorum
234. In acute diverticulitis of the colon, the
sigmoidoscopic finding is :
1. Mucosa is inflammed
2. Minute diverticuli seen
3. Saw toothed appearance
4. Sigmoidoscope cannot be passed beyond 15 cm
235. The organism which is commonly isolated from
the urine in high imperforate anus which is
indlcatlve ' of fistula is :
1. E.coli
2. Streptococcus
3. Proteus
4. Staphylococcus
236. Most common cause of lower GI bleed in India is:
1. Cancer rectosigmoid
2. Benign tumour

2014 NBE ONELINER


3. Non specific ulcer

4. Hemorrhoids

237. Painless lower GI bleed is seen in child with:


1. Meckel's diverticulum
2. Rectal polyp
3. Anal fissure
4. Acute Appendicitis
238. In case of 60/M with Ca colon true is :
1. Lesion on Lt. Side of the colon presents with features of
anemia
2. Mucinous carcinoma' has a good prognosis
3. Duke's A stage should receive adjuvant chemotherapy
4. Solitary liver metastasis is not a contraindication for
surgery
239. Massive colonic bleeding in a patient of
Diverticulosis is from :
1. Coeliac artery
2. Superior mesentric artery
3. Gastro duodenal artey
4. Interior mesenteric artery
240. Non true regarding solitary rectal ulcer :
1. 20% are multiple
2. Recurrent rectal prolapse is a Cause
3. Involves Posterior wall
4. Manage by digital reposition
241. A 3 year old child comes with a partial rectal
prolapse. Your treatment
1. Masterly inactivity
2. Digital reposition
3. Submucous phenol injection
4. Surgical excision of prolapsed mucosa.
242. The minimum safe distal margin for a Ca rectum
is :
1. 1.5cm
2. 2 cm
3. 2.5 cm
4. 5 cm
243. True regarding the internal sphincter of
canal canal is:
1. Circular muscles
2. Striated muscles
3. Voluntary muscle 4. Supplied by autonomic nerves
244. Ove ideal time after birth to do an
'Invertogram' test is :
1. Immediately at birth
2. 1 hour
3. 3 hours
4. 6 hours
245. Commonest cause of tubular inflammatory
stricture of the rectum is :
1. Iatrogenic
2. Endometriosis
3. Lymphogranuloma inguinale
4. Schistosomiasis

246. The maximum length of the rectum which can be


examined by a finger proctoscopy is :
1. 5 cm
2. 10 cm
3. 15 cm
4. 20 cm
247. Strawberry lesion of the rectosigmoid is caused
by:
1. Spirochaeta vincenti
2. Streptococcus
3. Gonococcus
4. Lymphogranuloma inguinale
248. All are parts of the anorectal ring, EXCEPT:
1. Puborectalis muscle
2. Superficial external sphincter
3. Deep external sphincter
4. Highest part of internal sphincter
249. The commonest location of an anal fissure is :
1. Midline posterior
2. Midline anterior
3. Left lateral
4. Right lateral
250. All are true regarding anal canal cancers
EXCEPT:
1. Usually squamous cancer
2. Lymphatic spread is to inguinal nodes
3. More prevalent in
251. Which is false about Insulinoma?
1. Mostly benign tumour
2. Weight loss
3. Hypoglycemic attacks
4. Usually solitary tumour
252. A patient with chronic pancreatitis gives "chain
of lakes" appearance in ERCP examination. Rx of
choice is :
1. Total pancreatectomy
2. Sphincteroplasty
3. Side to side pancreaticojejunostomy
4. Resecting tail of pancreas and performing
pancreaticojejunostomy
253. Regarding pancreatic trauma, false statement is:
1. Hyperamylasemia may occur without pancreatic
injury
2. Abdominal X-Rays are often not helpful in diagnosis
3. Commonest mode is penetrating trauma
4. ERCP is mandatory in all cases of abdominal trauma
254. Treatment of choice for symptomatic annular
pancreas is :
1. Resection of pancreas
2. Duodenoduodenostomy
3. Observation only
4. Gastrojejunostomy
255. One week after splenectomy for blunt
abdominal trauma a 36 year old female complains of
upper abdominal pain and lower chest pain which is

2014 NBE ONELINER


exacerbated by deep breath. She is ambulatory,
anorectic and making satisfactory progress. On
examination temp is 38.2C, decreased breath sounds
at left base. Abdominal wound is healing well and has
no signs of peritonitis . Rectal examination is
negative TLC is 12500/mm3 with left shift .
Abdominal X-Ray Shows non specific gas pattern .
Serum Amylase is 150 somogyi units. Most likely
diagnosis is :
1. Pulmonary embolism
2. Pancreatitis
3. Subphrenic abcess
3. Subfascial wound
infection
256. Most common tumor of pancreas is :
1. Insulinoma
2. Lipoma
3. Gastrinoma
4. Glucagonoma
257. Preferred management of Pancreatic abcess in
young male is :
1. Cystojejunostomy
2. Needle aspiration
3. External drainage
4. Cystogastrostomy
258. The treatment of choice for a mucocele of gall
bladder in young female is:
1. Antibiotics and observation 2. Cholecystectomy
3. Aspiration of mucous
4. Cholecystostomy
259. Which of the following types of pancreatitis has
the best prognosis:
1. Alcoholic pancreatitis
2. Gall stone pancreatitis
3. Post operative pancreatitis
4. Idiopathic pancreatitis
260. Incidence of gall stone is high in :
1. Partial hepatectomy
2. Ileal resection
3. Jejunal resection
4. Subtotal gastrectomy
261. Which one of the following is not a premalignant
condition for Ca gall bladder?
1. Typhoid carriers
2. Cholecystoenteric fistula
3. Rbrcelain gall bladder 4. Acalculous cholecystitis

4. Hepaticojejunostomy
264. Which of the following is not an indication for
cholecystectomy?
1. 40 year old male with symptomatic cholelithiasis
2. 15 year old male with sickle cell anemia and gall stones
3. 30 year old male with large gall bladder polyp
4. 25 year old with asymptomatic gallstones
265. Which of the following is not associated with
cholangio carcinoma:
1. Gall stones
2. Ulcerative colitis
3. Sclerosing cholangitis
4. Clonorchis sinensis
266. A patient presents with CBD stone of 2.5 cm 1
year after cholecystectomy. The treatment of choice
would be :
1. Supraduodenal choledochotomy
2. Transduodenal sphincterotomy
3. Endoscopic sphincterotomy with stone extraction
4. Transduodenal Choledochojejunostomy
267. Treatment of type I choledochal cyst in 2 year
old infant:
1. Cholecystectomy
2. Cholecystojejunostomy
3. Cystectomy
4. Excision and reconstruction with Roux loop .
268. Treatment for symptomatic retained CBD stones
in 40 year old female with cholecystectomy done 6
months ago:
1. Medical dissolution of stones
2. Conservative treatment with antibiotics
3. Immediate surgery
4. Endoscopic sphincterotomy
269. Contraindications for laparoscopic
cholecystectomy are all, EXCEPT:
1. Cirrhosis
2. Prior upper abdominal surgery
3. Suspected carcinoma gall bladder
4. Mucocle of gall bladder

262. Clinical features of choledochal cyst in child is :


1. Pain, Fever,Progressive Jaundice
2. Pain , Lump, Intermittent Jaundice
3. Pain , Lump,Progressive jaundice
4. Pain , Fever ,Intermittent Jaundice

270. After exploration of common bile duct, the TTube is removed on which of the following days :
1. 6 Postop day
2. 4 Postop day
3. 12 Postop day
4. 3 Postop Day

263. Chhaya a 28 year old lady developed bile


leakage on 51 post cholecystectomy day due to
lateral CBD tear. No T tube was inserted at the time of
surgery. The ideal treatment is :
1. ERCP and stenting
2. Primary repair after reopening abdomen
3. Ultrasound guided insertion of drain

271. Which of the following is false about splenic


artery aneurysms, EXCEPT:
1. Always have to be operated as mortality is high after
rupture
2. Occasionally cause bruit in left hypochondrium
3. Commoner in males
4. Calcified ring on left side of LV-1 on plain X-ray

2014 NBE ONELINER


272. Splenectomy is most useful in :
1. Thalassemia
2. Sickle cell anemia
3. Hereditary spherocytosis
4. Acquired autoimmune hemolytic anemia
273. A patient of hereditary spherocytosis
underwent splenectomy for the same. She was
asymptomatic for 2 years. After which she started
having recurrence of symptoms. USG does not show
any abnormal lesion in abdomen. You schedule her
for relaparotoglY. You expect to find the possible
cause of her problem at all the following sites ,
EXCEPT :
1. Porta hepatic
2. Splenic Ligaments
3. Mesocolon
4. Tail of pancreas
274. Insullinoma is most commonly located in which
part of the pancreas:
1. Head
2. Body
3. Tail
4. Equally distributed
275. The ideal treatment of stenosis of sphincter of
Oddi is:
1. Transduodenal sphincteroplasty
2. Endoscopic sphincterotomy
3. Choledochojejunostomy
4. Choledochoduodenostomy
276. All the following can be used for treating
femoral hernia, EXCEPT:
1. Lockwood operation
2. Lotheisen operation
3. Moloney's operation
4. Stopas operation
277. All contribute to spread of peritonitis, EXCEPT:
1. Adulthood
2. Ingestion of food
3. Steroid therapy
4. Use of enema
278. Most common age of presentation of patent
urachus is :
1. Birth
2. Infancy
3. Adolescence
4. Old age
279. About Bochdaleks hernia all of the following are
true, EXCEPT:
1. In early presentation, the prognosis is better
2. Ultrasound gives early antenatal diagnosis
3. Commonly contains colon and stomach
4. Left sided is commoner than the right
280. A 2 month old male child is brought to your
OPD. His mother has noticed a swelling near his
umbilicus which appears on crying and disappears
when the child is quiet. The child otherwise is

healthy and takes feeds regularly. You would


advise the mother to wait till the child grows upto :
1. 6 months
2. 1 year
3. 18 months
4. 2 years
281. Desmoid tumour occurs in :
1. Breast
2. Brain
3. Abdominal
4. Bone
282. Divarication of recti above the level of
umbilicus is seen in :
1. Babies
2. Elderly women
3. Multiparae
4. Malnourished men
283. Sister Joseph module at umbilicus is from:
1. Stomach
2. Ovary
3. Breast
4. All of the above
284. A 35 year old lady presents to surgery OPD
with pain and bleeding at the umbilicus. She gives
history of such episodes during her periods. The
most likely diagnosis would be :
1. Secondary carcinoma
2. Omphalitis
3. Endometrioma
4. Raspberry tumour
285. True statement about femoral hernia are all,
EXCEPT:
1. Occurs exclusively in females
2. Pregnancy is common cause
3. Strangulates
4. In males it is associated with cryptorchidism
286. Rasberry tumor is :
1. Neoplastic
2. Inflammatory
3. Traumatic
4. Congential
287. Regarding to familial Mediterranean fever the
true statement is , all , EXCEPT:
1. It manifest as a periodic disease with complete
remissions in between
2. Commoner in females
3. Peritoneum around gall bladder and spleen is inflamed
4. Colchicine is the causative agent
288. Prevescial hernia is :
1. A type of direct hernia
2. A type of indirect hernia
3. A type of femoral hernia
4. Same as Spigelian hernia
289. Progressive bacterial synergistic gangrene is
usually seen after:
1. Colostomy for perforated colon
2. Cholecystectomy
3. Colonic surgery
4. Laparotomy for a perforated viscus

2014 NBE ONELINER


290. The most common presentation of chronic
tuberculous peritonitis is:
1. Loss of weight
2. Fever
3. Pain abdomen
4. Ascites
291. Exomphalos minor is a disease involving:
1. Diaphragm
2. Cervix
3. Abdominal wall
4. Urinary bladder
292 .The pathognomonic sign of impending burst
abdomen is:
1. Shock
2. Pain
3. Serous sanguineous discharge
4. Signs of intestinal obstruction
293. Treatment for pneumoperitoneum due to
perforation during colonoscopy is :
1. Temporary colostomy
2. Closure & lavage
3. Symptomatic
4. Permanent colostomy
294. Most common organism causing omphalitis is:
1. Staphylococcus
2. Streptococcus
3. E-coli
4. Clostridium tetani
295. A 45 year old male develops pain in right lower
abdomen while playing squash. The pain progresses
and she presents to the emergency room later that
day with low grade fever, while blood cell count of
20,000 and complaints of anorexia, nausea and
persistent sharp pain in right lower quadrant. On
examination he is tender, in right lower quadrant,
with muscular spasm and there is a feel of mass
effect. On ultrasound a mass in the abdominal wall is
seen. The most likely diagnosis is :
1. Torsion of an ovarian cyst
2. Haematoma of rectus sheath
3. Acute appendicitis
4. Strangulated Spighelian hernia
296. Colpotomy is done to treat:
1. Ischiorectal abscess
2. Pelvic abscess
3. Appendicular abscess
4. Perianal abscess
297. Most common cause for development of
incisional hernia is :
1. Obesity
2. Post operative cough
3. Prolonged ileus
4. Post operative wound infection

300. Incorrect about obturator hernia is :


1. Commoner in males
2. Commoner in people > 60 year of age
3. PV/PR examination may
reveal a tender swelling
4. Is usually Richter's type of hernia

1. (3)
Following infection by the HIV-1 virus into the blood, there is a
brief seroconversjon illness that is characterized by flu-like
illness & lymphadenopathy. There then follows a latent period
when the infected subject appears well but has a progressive
fall in CD4 counts. In this phase, there is actually a fall in viable
HIV virus in the body that subsequently rises during
development of AIDS. Systemic immune deficiency occurs
during. AIDS phase of illness.
2. (1)
Risk with solid-needle is 10-fold less than with hollow-needle.
Post-exposure HIV prophylaxis should be offered when the
source patient from high-risk group & his HIV status is
unknown. As it should be started within 1 hour when possible
it is inappropriate to wait to know HIV status. 3 drug
prophylaxis (Zidovudine, Lamivudine & indinavir) is given for
1 month.
3. (2)
It is the most important aspect of treating wound abscess,
whether antibiotics are used or not. Antibiotics usage is
controversial unless cellulitis, lymphangitis or related sepsis is
suspected. (3) & (4) are associated with poor healing
of wound abscess.
4. (3)
Cellulitis is a poorly localized, non-suppurative invasive
infection of tissues. All cardinal signs of inflammation are
present. Blood cultures are often negative, but SIRS is common.
Lymphangitis presents with painful red streaks & painful
enlarged lymph nodes.
Abscess is well-localised.
SSSI is superficial surgical site infection or an infected wound.
5. (1 )
A major wound infection is defined as a wound that either
discharges significant quantities of pus spontaneously or needs
a secondary procedure to drain it (3) & (4) may also be
associated.
Minor wound infections will also discharge pus or infected
serous fluid, but is not associated with (2), (3) and (4).

298. Abdominal dehiscence is common on:


2. 4th post operative day
1. 2nd post operative day
3. 6th post operative day
4. 12th post operative day

6. (2)
Most abscesses take 7-9 days to form after surgery. As many as
75% of infections may present after the patient has
left hospital & thus, may be overlooked by the surgical team.

299. Lytles method is used for preparing:


1. Indirect inguinal hernia
2. Direct Inguinal hernia
3. Femoral hernia
4. Umbilical hernia

7. (3)
Control of air quality in a modern OT is important because
non-visceral bacterial contamination of the wounds is
predominantly from the air in OT. (1), (2) and (4) are all

2014 NBE ONELINER


effective methods to reduce post-operative air-borne infection.
Ultra-clean air in OT should have a bacterial count of less than
10 CFUs per cubic meter.
8. (3)
One application of any alcoholic antiseptic is adequate for skin
preparation & reduces the bacterial count by more than 95%.
Cetrimide (Savlon) has an aqueous presentation &. is useful in
hand-washing & instrument & surface cleaning.
9. (2)
SIRS includes any 2 of following (1) Hyperthermia (> 38C) or
hypothermia < 36C.
Tachycardia> 90. min or tachypnoea> 20 min.
WBC < 4000 or> 12,000/cmm
Sepsis is SIRS with documented infection
Sepsis syndrome is sepsis with incidence of one or more organ
failure like ARDSI renal failurel coagulation
abnormality.
10. (1)
It affects younger age groups and alcohol does not playa part in
the etiology, Ass with ingestion of cassaral Tapoica esp in
southern India due to inability to detoxify cyanogens as a
result of malnutrition. Diagnosed by CT/USG with finding of
nodular fibrotic pancreas with dilated stone filled ducts.
Management is mainly medical with surgical Rx for
intractable pain (side to side pancreatojejunostomy done).
11. (3)
Typhoid is gram negative bacillus infection affecting peyers
patches of small intestines leading to necrosis and ulcerations.
If untreated may lead to malena secondary to bleeding or
perforate (usually 2nd 13rd weeks). Diagnosis can be done by
Widal (2nd week)1 Blood or stool culture (1st week)1 urine
culture (3rd week). Usual Rx is medical with metronidazole I
cephalosporin's and gentamicin. Surgery is required in patients
with perforative peritonitis. Patient in 1st week of ulceration if
perforates has better prognosis as in early phase patient is less
nutritionally compromised and body defence robust also
better prognosis due to shorter interval between diagnosis and
operation.
12. (1)
First scrub should be about 5 mins. Usually chlorhexidine used
(acts> 4 hrs)fuan iodine (action < 4 hrs). Ideally 20 air
charges/hr with a 5 mm pore size filter is a adequate for a
general surgical theatre. Cotton suits below cotton gowns
decrease bacterial count by 30% in air and by 47% if elastic
anklets used ovser the trousers.
13. (3)
Dubois abcess are thymus gland abscess in congenital syphilis.
Conlis also causes sabretibia, Hutichinsons
teeth, Moons molars, Cluttons joints. Mycoticabcess are due to
fungal infection.
14.
15.
16. (3)

Commonest cause of acute LNadenitis is staphylococcal skin


infection and chronic LNadenitis is TB.
17. (1 )
Tetanus is caused by CI. tetani.
Gas gangrene is caused by CI. Welchi / oedematieus/ septicuss.
18. (4)
The patient is 65 kg and requirement of protein is 2
gm/kg/day that is 130 gms/day
25% protein means 25 gm per 100 ml. solution.
That is 3000 ml solution contains 750 gm of protein.
Hence excess of protein is being transfused to this patient.
19. (2)
Hypoglycemia typically occurs during cessation of TPN and
results from raised plasma insulin levels which lag behind.
falling plasma glucose concentrations. This can be prevented
by gradual reduction in the rate of TPN, rather that abrupt
cessation. (1), (3), (4) may occur during TPN.
20. (4)
Epidural analgesia is approaching a gold standard in post
operative pain relief. Its use is associated with marked
reduction in mortality. They can remain in situ for 4 - 5 days
and provide excellent analgesia with either low concentration
local anaesthetics or opiates like diamorphine.
21. (2)
Metabolic features of starvation
Low plasma insulin concentrations
High plasma glucagon concentrations
Hepatic glycogenolysis.
Protein catabolism
Hepatic gluconeogenesis
Lipolysis: mobilization of fat stores.
Adaptive ketogenesis: Spares protein
Fall in energy requirements (to - 15 kcal kg-1 per day)
22. (4)
Level - I : Best evidence based on metanalysis and multiple well
designed controlled studies. Level II to IV - Evidence available
but not so strong
Level V - Evidence least compeling and based on case reports
and clinical examples. "A" grade recommendation is Best
evidence and "D" grade is weakest evidence.
23. (2)
Person ideally should be the surgeon who will carry out the
treatment and may not have enough understanding to counsel
the patient properly.
24. (3)
Metabolic features of starvation includes:
(a) Low plasma insulin concentrations.
(b) High g!ucagons levels.
(c) Hepatic glycogenolysis.
(d) Hepatic gluconeogenesis.
(e) Lipolysis.
(f) Fall in energy requirement up to 15 Kcal/ Kg day.
(g) Adaptive ketogenesis.

2014 NBE ONELINER


25. (4)
Nutritional assessment includes
Fat: Skin fold thickness triceps. Biceps / Ant abdo wall.

It will be inferior in function and cosmesis. A delto pectoral flap


is useful for external skin defects and not inner mucosal
defects.

26. (4)
90% of malignant melanomas arise from junctional naevi. They
are likely to undergo proliferation from time to time.

34. (3)
A deltopectoral flap has only skin and fascia hence it is a
fasciocutaneous flap. It is based on the 2nd/3rd perforator
branches of internal mammary artery and used for head &
neck reconstructions.

27. (4)
Malignant transformation of neurofibromatosis occurs in 5 10%. It is also called von Recklinghausen's disease
It is MC Hereditary Neurocutaneous syndrome
2 forms NF-1-Lisch nodules /cafe all lait spots
NF-2- Bilat vestibular Schwannomas.
Both Autosomal dominant
Neurofibromas differ from Schwanomas in that
They are unencapsulated benign neoplasms of Schwann cells
and fibro blasts Tumor involves nerve and hence nerve needs
to be sacrificed.
28. (4)
Neuroblastoma, Retinoblastoma/RCC & malignant melanomas
have shown spontaneous regression in few cases.
29. (4)
Congo torticollis is presents at birth causes are
Sternomastoid tumor (mc)
Pterygium coli
Klippel feil syndrome.
Hemivertebrae
Usually associated with Breech deliveries leading to injury to
Sternocleidomastoid mastoid muscle
Rx includes -passive stretching, exercises and splinting
Surgical release is used of consecutive Rx fails.
30. (1)
Epulis - Dental anomaly
Syphilis Infection T.palidum
Ephilis is a type of freckle
31. (3)
If sqcellca < 2cms = 4mm margins required
> 2 cms == 1 cm margins
If malignant melanoma < 1 mm deep =1cm margin.
Deeplesions = 2cm margin.
These little data to support use of margin under than 2 cms
If BCC, margins b/w 2-15 mm depends on macroscopic variant
32. (4)
Full thickness grafts give a good color match and texture and
hence good cosmesis. Only small areas can be grafted by full
thickness graft.
The scrotum back and scalp do not require cosmetic grafts;
hence split thickness is better. Full thickness is
commonly used for the face.
33. (4)
A free fibula flap will give the best cosrnesis and function.
Skin graft is not an option as it will not give any tissue bulk and
the contour is not even. A pectoralis major myocutaneous flap
is an option if a free fibula is not possible.

35. (3)
It derives the blood supply from the Deep inferior epigastric
artery and hence is also called the DlEA flap.
It can be used as a pedicled or free flap for breast
reconstruction.
36. (2)
For a skin graft (Full thickness or split thickness or amniotic
membrane) to survive it must be revascularized by recipient
bed. Radiation damaged tissues and relatively avascular
structure such as bone, tendon, cartilages are therefore poor
recipient sites.
So an exposed bone surface is covered by a graft which has its
own blood supply. Such grafts are k/a flaps or pedicle grafts.
Sabiston writes - "Pedicle graft or flap is a partially or
completely isolated segment of tissue with its own blood
supply"
"Absolute indications for flapsExposed bone, radiated vessels, brain, an open joint or
nonbiological implant materials. Pressure sores where a bony
prominence isexposed."
37. 4
Skin
substitutes

Advantage

Disadvantage

1. Cultured
allogenic
Keratinocytes
graft

Off the shelf


No Biopsy
needed
Provides wound
coverage
Provides healing

2.
Bioengineered
dermal
replacements
3. Cultured
Bilayer skin
equivalent

Prevents
contracture
Good re aration
for raft a lication
More closely
mimics normal
anatomy
Does not need 2
procedure
Easil handled,
sutured or meshed

unstable
Doesn't prevent
wound contracture
Fragile
Possible of disease
transmission
Inadequate cosmesis
Temporary
dressings
Decreased
reepithelisation
Cost
Short half life
True en raftment
questionable

38. (4)
Capillary Hemangiomas are
1) Salman patch - Present since birth
- Forehead or occiput
- Disappears by age 1 year
2) Portwinestains - diffuse telengaotenia, no swelling
- Face/lips/Buccal mucosa presense since
Birth

2014 NBE ONELINER


- No tendency to disappear
3) Strawberry angioma - Bright red/dark red raised Hair
hemispherical swelling usually on
head/neck & resembles a
strawberry
- Appears after birth grows till 1st
year then resolves by 7-8th year.
4) Spidernomus
- Usually acquired semina preg/t
liver cirhosis.
39. (4)
Clinical testing for brainstem death Absence of cranial nerve
reflexes
Pupillary reflex
Corneal reflex
Pharyngeal (gag) and tracheal (cough) reflex
Occulovestibular (caloic) reflex
Absence of motor response
The absence of a motor response to painful stimuli applied to
the head/face and the absence of a motor response within the
cranial nerve distribution to adequate stimulation of any
somatic area is an indicator of brainstem death.
The presence of spinal reflexes does not preclude
Absence of spontaneous respiration:
After pre-ventilation with 100% oxygen for atleast 5 min, the
patient is disconnected from the ventilator for 10 min to
confirm absence of respiratory effort, during which time the
arterial Pco, level should be > 8 kPa (60 mmHg) to ensure
adequate respiratory stimulation.
To prevent hypoxia during the apnoeic period, oxygen (6 I
min") is delivered via endotracheal catheter.
And test performed by 2 clinicians on two separate occasions
with atleast one is a consultant and none
associated with the transplant team.
40. (2)
Kidney
Liver
Pancreas
Small intestine
Heart
Lunq
41. (4)
Gas
C02

N 2O

Max storage
48
time (hrs)
24
24
8
6
8

Optimal storage time


<
24
(hrs)
.< 12
< 10
<4
<3
<3

Advantage
Low risk of venous gas
embolus Does not support
combustion
Insignificant acid-base
change, Decreased pain

Helium

Minimal effect on acid


base balance

Argon

Stableacid -base balance

Disadvantage
Hypercarbia and
acidosis
support combustion
in the presence of H2
or CH3 gas
Risk of venous
embolism subacute
emphysema
Cardiac depression

42. (2)
In suffilator used to monitor the intra abdominal pressure and
insufflate gas. Usually pressure set between 12-15 mm kg as
high pressures cause hypercarbia/acidosis and adverse
cardiorespiratory problems.

43. (4)
Principle causes of death in Renal transplant patients in
decreasing order is :
a) Heart disease
b) Infection
c) Stroke
Infection is usually viral origin commonly CMV.
44. (3)
Types of Graft Reiection
Types
Hyperacute

Time
Immediately

Acute

1st 6
months

Chronic

After 6
months

Cause
Preformed
antibodies
T cell De en dent
immune
response
Non immune
factors

HP
Intravascular
thrombosis
Mononuclear cell
infiltration
Myointimal
proliferation of
graft vessels

46. (3)
The transplant kidney is placed in the iliac fossa in
retroperitoneal position living the native kidney in situ. Donor
Renal vein is anastomosed to Ext iliac vein. The donor renal A
is anastomosed end to side to ext iliac artery. If donor Arter:y
lacks a Carel's patch like in living donor, it is anastomosed to
intiliac artery end to end.
47. (3)
NOTES is natural orifice transluminal endoscopic surgery. It's a
technique whereby the peritoneal cavity is entered
endoscopically with natural orifices like month, anus or vagina
and surgery done ego Cholecystectomy and
appendicectomy.
48. (3)
Triangle of doom is bound by vas deferens, testicular vessels
and reflected part of peritoneal folds. It is a danger area of no
tackers/sutures or dissection in lap Hemia Surgery. It contains
iliac vessels.
49. (4)
Variable length is 18-45 cms but regularly used instruments
are 36 ems in adults and 28 cms for Paediatrics patients. 18
cms to 25 ems for cervical cases and paediatric.
45 cms = obese /very tall points.
Vessels needle used is b/w 80- 120 mm in length.
80 mm - thin patients, 120 mm for obese patients.
50. (4)
Donor-type lymphoid cells transplanted within a graft may
recognize the host's tissue as foreign and mount an immune
response against the host. This response, termed graft versus
host disease (GVHD), is common in bone marrow
transplantation and is an important source of morbidity and
mortality. Treatment requires more aggressive immuno
suppression. Current clinical practice includes depletion of
lymphocytes from the marrow graft in order to prevent the
development of GVHD, GVHD has been documented following
liver transplantation, presumably because of the large amount
of lymphoid tissue in the donor liver. GVHD has not been

2014 NBE ONELINER


described following heart, lung pancreas or kidney
transplantation.

51. (2)
Congenital lymphedema is usually bilateral. It affects lower
limbs more often and commonly occurs before puberty.
Repeated lymphangitis causes obliteration of the already
deficient lymphatics and edema is worsened.
52. (3)
Topical antibiotics are ineffective in healing leg ulcers and are
particularly likely to produce skin sensitization. They should
never be used in the management of venous ulceration.
Patients who have eczematous reactions around their ulcers
may require the use of topical steroids to treat the allergic
response.
53. (4)
The use of graduated compression stockings, exercise during
the flight and the avoidance of alcohol and sleeping tablets is
probably all that is required. In particularly high risk
passengers, that is those with a previous history of venous
thromboembolism, recent surgery or strong family history of
thrombosis, low-molecular weight heparin can be
administered subcutaneously before the flight.
54. (3)
The commonest Iymphangiographic finding in lymphoedema
praecox is distal obliteration with absent or reduced
distal superficial lymphatics. It is also termed aplasia or
hypoplasia.
55. (3)
(1), (2) and (4) are true statements. DEC destroys the parasites
but does not reverse the lymphatic changes,
although there may be some regression over time.
56. (2)
Isotope Iympho-scintigraphy has now largely replaced
lymphangiography, which was previously considered the gold
standard. Radioactive Tc-labelled protein or colloid particles
are injected into an interdigital web space and specifically
taken up by lymphatics, and serial radiographs are taken with
a gamma-camera.
57. (4)
Management of primary lymphoedema required a holistic
approach. The current preferred term 'decongestive
lymphoedema therapy' (DL T) includes 2 phases.
(1) 1st intensive phase (therapist -led) consisting of (1), (2),
(3) and exercises.
(2) 2nd maintenance phase (self -care)
Diuretics are of no value in pure lymphoedema.
58. (2)
Filariasis is the commonest cause of chyluria occurring in 1-2%
of cases 10-20 years after initial infestation. It presents as
painless passage of milky white urine, particularly after a fatty
meal. It may also be caused by ascariasis,
Malaria, tumour and tuberculosis.

59. (3)
Aetioloaical classification of! Mohoedema
Primary
Congenital (onset <2 Years old); sporadic;
lymphoedema
familial (Nonne-Milroy's disease) Praecox
(onset 2-35 Years old):
Sporadic; familial (Iatessier - Meige's
disease)
Tarda (onset after 35 years old)
Secondary
lymphoedema

Parasitic infection(filariasis)
Fugal infection (tinea pedis)
Exposure to foreign body material (silica
particles)
Primary lymphatic malignancy
Metastatic spread to lymph nodes
Surgical excision of lymph nodes
Trauma (particularly degloving injuries)
Superficial thrombophlebitis
Deep venous thrombosis
Stewart Treve's syndrome is Iyrnphagiosarcoma in chronic
lymphadernatous limb.
60. (2)
Alemtuzumab Antibody agent against CD52 antigen - Rx of
lymphoma.
Rituximab - Antibody against CD20 antigen - Rx of lymphoma.
Tyrosine kinases receptors inhibitors - Imatinib - Rx of GIST.
Gefitinib/Erlotinib - Rx - Non small cell lung tumors
Farnesyl transferase inhibitors = Lonafarnibl Tipifarnib Against leukaemia
61. (2)
Severity of unilateral lymphoedema is classified as. Mild - <
20% excess volume
Mod -20-40%
Severe - > 40%
62. (2)
Bad prognosis in Hodgkin's disease seen with
- Albumin < 4 gm %
- Age> 45 yrs
- Hb < 10.5 gm%
- WBC 15,000/cm
- Male
- Stage IV disease
- lymphocyte count < 600/l or 8% of WBC.
63. (4)
Working formulation for NHL is
Low grade
- Small cell lymphocyte
- Follicular small cleaved cues.
- Follicular small and large cleaved cells.
Int grade
- Follicular large cell - Diffuse small cell
- Diffuse large cell
- Diffuse mixed cell
High grade
- Lymphoblastic
- Immunoblastic
- Small non cleaved cells.
Option a, c, d - come under Integrade. b is low grade. Of int
grade, most aggressive is diffuse large cell type.
64. (2)

2014 NBE ONELINER


Endemic elephantiasis is also called as podocaniosis common
in tropics. Where people walk bare foot in alkaline volcanic
rocks in Africa causing destruction of peripheral lymphatics by
silica particles. Plaster oedema develop is childhood and
rapidly spreads proximally. Condition is prevented and
progression slowed by wearing shoes.
65. (4)
Grade
Clinical features
(Brunner)
Subclinical There is excess interstitial fluid and Histological
(latent)
abnormaiities in lymphatics and lymph nodes,
but no clinicall apparent lymphoedema
I
Oedema pits on pressure and swelling largely or
completely disappears on elevation and bed rest
II
Oedema does not it and does not significantly
reduce upon elevation
III
Oedema is associated with irreversible skin
changes that is fibrosis, papiIIae
66. (1)
Cystic hygroma is seen in young children / infants with
swelling increasing on crying with positive fluctuation,
transillumination and compressibility it is a lymphangioma;
Excision of all sacs is the treatment.
Solitary lymph cyst - though congenital, Present is adult life. It
is a single cyst containing lymph develops in same
process is cystic hygroma.
Branchial cyst - Cystic swelling due to persistence of cervical
sinus although congenital, appears first around 20-25 year age,
lump is below angle of mandible. Fluctuation,
transillumination negative, Rx excision.
67. (1)
Ulceration and gangrene occur with severe arterial
insufficiency and often presents as a painful superficial erosion
between the toes. (3) is seen in moderate as well as severe
ischaemia and limbs become blanched on elevation and
develop a purple discolouration on dependency.
Rest pain in severe pain felt in the foot at rest made worse by
lying down or elevation of the foot.
Pain in calf on walking is seen in intermittent claudication. This
is relieved by standing still. As the degree of ischemia increases
from I-III, the claudication distance decreases and it progresses
towards rest pain.
68. (4)
Placement in the Trendelenburg position encourages air to
pass into the veins of the lower half of the body. Placinq the
patient on the left side will allow air to float into the apex of the
ventricle away from the pulmonary artery.
Oxygen is administered to counteract hypoxemia and to assist
in the excretion of nitrogen. In serious cases, the right ventricle
should be aspirated by a needle passed upwards and
backwards from below the left costal margin.
69. (3)
One of the earliest signs may be emboli in the retinal arteries,
which cause striate haemorrhages and 'fluffy' patches of
exudates. (1), (2) and (4) are true.
70. (1)

Sympathectomy is of no value in intermittent claudication


Possible indications for sympathectomy
Ischemia disorders mainly of the limbs
- Buerger's ds.
- Raynaud's ds
- Arterial injury or embolism
- Acrocyanosis
- Erythrocyanosis
- Frost bite
Atherosclerotic ischemia of limb.
71. (4)
An ABI of less than 0.9 suggest arterial injury even in the
presence of palpable pulses in patients with penetrating
injuries. This will prompt early arteriography and allow for a
successful early surgical intervention.
72. (3)
X-ray may show widened mediastinum but the gold standard
for diagnosis and of aortic rupture is the aortogram but a
multi slice CT scan with contrast yields similar results.
73. (3)
In BK amputation following considerations to be taken :
a) Nerves ligated higher up after pulling them down
b) Post flap longer than ant flap.
c) Stump should be of adequate length not too short
d) Margins and Scar healthy central oranterior.
e) No dog earsl Bony spurts.
f) Fibula transected 2 cms above the tibial
74. (4)
Aortic dissection
- Most common affects ascending aorta
- man 50 - 70 yrs age, Afro- caribbeans.
- Associated with HTN /marfans/preg/Co-arctation of aortic. Severe tearing intrascapular pain.
- Suspected on in X-ray chest seeing widened mediastinum.
- Traditionally aorto graphy is gold standard but can be picked
up by 2D ECHO/CT/MRI also.
- 2 types A - involves ascending aorta.
B - does not involve ascending aorta.
Rx - Type A - Always surgery.
Type B - medical Rx with control of Bp.
75. (1)
Temporary Av fistula called cimino fistula noted at wrist
between radial A and cephalic vein.
If at ankle it is between post tibial A and near by vein.
76. (3)
Palate is the most common site of ectopic/minor salivary gland
tumor. 80% of these tumor are malignant. Rx of these tumours
is surgical excision.
77. (2)
Pain is not a reliable indicator malignancy as benign tumours
often present with pain & aching in the affected gland,
presumably due to capsular distension or outflow obstruction.
Besides (1), (3) & (4), induration and\or ulceration of
overlying skin is also a clinical feature of malignancy.

2014 NBE ONELINER


78. (3)
Warthin's tumour is a benign tumour arising most commonly
from the parotid gland. It produces hot pertechnetate scan.
Prognosis is good. Treatment of this tumour is superficial
parotidectomy.
79. (3)
This classical picture occurs when the stone causes almost
complete obstruction, usually at the opening of the
submandibular duct. In (1) & (2) the symptoms are more in
frequent, producing minimal discomfort & swelling, which is
not confined to mealtimes.
80. (2)
The most common organism to cause acute bacterial parotitis
is S. aureus. Streptococci, bacteroides and other
anaerobe organisms are also implicated.
81. (2)
(1), (3) & (4) are all used as methods for prevention of Frey's
syndrome as they are local flaps.
Frey's syndrome results from damage of the innervation of the
salivary gland during dissection in parotidectomy surgery.
There is inappropriate regeneration of parasympathetic
autonomic nerve fibres which thus stimulate the sweat glands
of the overlying skin.
82. (4)
This represents metastasis in a jugulo digastric node. This is
squamous cell carcinoma. (2), (3) & conventional excision are
required. Recurrent disease is managed by surgery &
microvascular free flap reconstruction. Neck dissection is
required in a large proportion of cases of advanced disease.

N2a = Mets in single ipsilateral Ln size > 3 cms < 6 cms.


N2b = multiple LN mets in ipsilateral none > 6 cms
N2C = mets in Bilateral or contralateral Ln none> 6 cms.
N3 = Mets in LN > 6 cms size.
86. (3)
50-65% of cystic hygroma are present at birth they are due to
sequestration of a portion of jugular lymph sacs.
Sternomastoid tumour - Injury to sterno mastoid muscle
during delivery leading to fibrosis and contracture with
palpable mass palpated soon after birth.
87. (3)
- It is also called potato tumor or chemodectoma.
- Non chromaffin paraganglioma ass with pheochromocytoma.
- Arises from chemoreceptor cells from carotid Bulb.
- Usually unilateral.
- 10% cases have &.H/0 familial predisposition and affects
people in 5th decade.
- Slowly enlarging painless lump firm rubbery, pulsatile mobile
form side to side but not up down.
- Confirmed by Doppler and CT angio. FNAC and biopsy are
contraindicated.
- Rarely metastasis.
- Rx- Surgical excision If sized> 2 cms preop embolisation is
helpful.
88. (1)
It consists of - sq cells! mucus secreting of cells:
Inf. Hybrid cells and clear or Hydropic cells.
Mixed parotid Ca caused by myoepithelial cells (Epithelial +
mesenchymal)
39. (4)

83. (3)
Rx for chronic lymphoedema includes charleslThompson and
Iympho venous anastomosis surgeries.
Sx for cleft lip is include millard's Rx, Lemesurier's Sx and
Tennessean's Sx.
Sx for cleft palate include Hynes and Wardills pharyngoplasty.
Newman's' and SeaBrock's Sx is for repair and parotid fistula.
84. (3)
Lymph mets of buccal Ca goes to sub mandibular and upper
deep central nodes.
Ca lip spreads to sub mental LNS; also form tip of tongue.
Bilateral lymphatic spread is common in following tumors of
the head and neck
Lower lip Supraglottis Soft palate
85. (3)
TNM staging for oropharyngeal Ca includes
T = No evidence of 1 tumor
T5 = Ca insitu
T1 = < 2 cms size
T2 = 2 - 4 cms size
T3 = > 4 cms size
T4 = involves adjoining structures!
Muscles, Bones
Nx = lymph nodes cannot be assessed
N0 = No LN mets
N1 = Mets in single ipsilateral size <3 cms.

89. (4)
Medical treatment with antifungal drugs may be necessary for
many months to eliminate the organisms & reinfection is a
constant problem. Surgical excision is recommended for
persistent lesions.
90. (2)
Oral submucous fibrosis is not in itself premalignant but is
associated with a higher than normal incidence of oral cancer.
(1) & (3) are conditions about which there is doubt as to
whether their association with oral cancer is causal or casual.
(4) is a premalignant condition.
91. (4)
In contrast with mandibular alveolar tumours, deep infiltration
into the underlying bone is uncommon. Reverse smoking is
responsible for palatal cancers. Most of them arise from minor
salivary gland.
92. (2)
MRI is investigation of choice for oropharyngeal cancer. The
advantage over CT is that the image is not degraded by the
presence of metallic dental restorations. It is very good at
imaging soft tissue infiltration.
Plain radiography is of limited value in investigation of oral
cancer. At least 50% of the calcified component of bone must
be lost before any radiographic change is apparent.

2014 NBE ONELINER


Radionuclide studies are of limited value in diagnosis of
primary oral cancers.
93. (4)
Besides these three, radial forearm flap & rectus abdominus
flap can also be used for reconstruction.
94. (4)
All of the above are techniques followed for primary
reconstruction of the mandible.
95. (1)
Invasion in the edentulous mandible is almost exclusively via
deficiencies in the cortical bone of the alveolar crest. In the
dentate mandible, invasion is usually via the periodontal
ligament & is nearly always above the insertion of the
mylohyoid muscle. Once the tumour has invaded the mandible,
it soon enters the inferior dental canal & perineural spread
occurs anteriorly & posteriorly.
96. (1)
Commonest variety is undifferentiated squamous cell
carcinoma. Only few lessions are well differentiated squamous
cell carcinoma & these two lesions make up over 90% of
nasopharyngeal malignancy in endemic areas.
Rest all are true.
97. (1)
- non chromaffin producing paragangliomas
- middle aged females affected
- Non encapsulated, extremely vascular and locally invasive
tumour but can metastizes to the lung/liver/ LNS
- Usually arise from adventitia of jugular bulb (gl. Jugular) or
Jackson's N in temporal bone (gl. tympanicum).
- Earliest symptoms is deafness (conductive with pulsatile
tinnitus)
- Rising sum appearance on otoscopy.
- Angiography provides a definitive diagnosis
- CT shows salt paper appearance of involved base.
- TOC is surgery with preop embolisation
98. (1)
Rx of HFN tumors when there is involvement of cervical lymph
nodes, the 10 and nodes are both treated surgically. The gold
standard has been radical neck dissection, all modification of
neck dissections are described in relation to std radical neck
dissection.
Rx of Ca tongue size < 1cms = Sx removal with wide margins
large mass - RT is 10 Rx and Sx reserved for salvage. Growth
reaches with in 2 cms of jaw Sx is 1 mode with pre or post
op RT as adjuvant Rx.
Reconstruction required if defect.
< 1/3rd tongue - Not required
> 1/3rd < 2/3rd tongue - radial forearm flap
> 2/3rd tongue - pectoralis major flap.
99. (1)
5 yr. survival rates
Site
Lip
Stage I
90%
and II
Stage III
50%

Tongue
70%

Palate
80%

Check
65-75%

40%

40%

30-50%

and IV
So for same stage, Ca lip has highest 5 year survival rate or
have best prognosis.
100. (3)
Maxillary Ca is two types:
(a) Sq cell Ca (m.c. type)
(b) Adeno ca
Rx of Ca maxilla is
Sq cell ca -7 combination of Sx and RT gives better results than
either Rx alone. (Rx is same in every stage of maxillary Ca).
Adena ca - only Sx done as RT is in effective.
101. (4)
MEN - II syndromes are due to mutation in RET.
Protooncogene present on chromosome 10,
MEN IIA - Medullary thyroid cancer, Pheochromocytoma
hyperparathyroidism.
MEN II B - Medullary thyroid cancer. Pheochromocytoma.
Mucosal neuromas, megacolon, skeletal abnormalities.
102. (4)
Excision of both adrenal glands
Schwartz
"Patients undergoing surgical treatment of endogenous
hypercortisolism require glucocorticoid replacement steroids
are not given preoperatively because these patients are
already hypercortisolemic. Instead hydrocortisone 100 mg i.v,
is given after the removal of the~cor}qhYperplastic adrenal
gland.
C.S.D.T.
"After total adrenalectomy, life long corticosteroid
maintenance theraphy becomes necessary. The following
schedule is commonly used. No cortisol is given until the
adrenals are removed during surgery.
On the firest day 100 mg iv is given every 8 hours.
Love & Bailey
According to L&B, 23rd /e corticosteroid replacement is
necessary during both intraoperative and postoperative while
according to 24th/e corticosteroid therapy intraoperative is
only needed when the patient has been treated with
Ketoconazole or mitotane.
L&B 23rd /e
"It is essential that all patients who are to be subjected to
adrenalectomy are supported intraoperatively and
postoperatively by adrenocortical hormone replacement
therapy irrespective of the extent of adrenal resection ,
corticosteroids are started when anaesthesia is induced,"
L&B 24th
" In the new edition they have removed these lines and states
that
Only patients who have received medical therapy with
mitotane and Ketoconazole should be given
Corticosteroid at the induction of anaesthesia and also adds
that "All patients who have undergone adrenal surgery
(U/L or B/L) will require Post operative steroid replacement.
103. (4)
All the given options can cause Cushing's syndrome. However
exogenous administration of steroids for treatment is most
common cause of Cushing's syndrome. Most common

2014 NBE ONELINER


endogenous cause for Cushing's syndrome is pituitary
dependant Cushing's disease.
104. (3)
Autoimmune disease is responsible for 60% of cases of
Addison's disease. Remaining options viz infections,
amyloidosis and metastases constitute remaining 40%.
105. (3)
Fluids are used to preload the patient for him to withstand
virtual hypotension caused by removal of tumour, propranolol
is used to initiate blockade, phenoxybenzamine is
irreversible a blocker is first drug of choice for bringing down
BP in these patients. Nifedepine has no role in preoperative
preparation of these patients.
106. (2)
Ad renal Incidentalomas are clinically unapparent masses
detected accidentally by imaging studies done for other reason.
They are defected in 4% cafes and prevalence Increases with
age.
- More than 75% cases are non functional adenomas only 4%
are Pheochromocytomas
- Complete Hist/Examn and biochemical work up is required
with CT/MRI.
- Hormonal evaluation includes 24hrs excretion of the
catecholamine, metanephrines, 1mg overnight dexamethasone
suppression test, SrK+, aldosterone & renin activity Sr DHEAS,
testosterone or 17--OH estradiol.
- FNAC done after phaeochromocytoma is ruled out
- Rx smaller non functional masses <4 cms -following up with
CT/Hormone analysis at 6, 12, 24 mths. Mass > 4cms /
functional mass are excised.
107. (4)
The difference between malignant and benign tumors is
difficult except if mets are present. An increased PASS
phaeochromocytoma mass of phaeochromocytoma of the
adrenal gland scale score indicates malignancy as does a high
number of Ki - 67 positive cells, vascular invasion or a
breached capsule. A malignant secretes excess of
Noradrenaline than adrenaline whereas benign have higher
adrenaline levels. .
108. (3)
- Craniopharyngiomas are cystic tumors with area of
calcification
- Originate in epithelial segments of rathke's pouch
- Usually benign mass found in Sellar and suprasellar region
leading to Compression of pituitary optic tracts and third
ventricle.
- Radio graph shows area of sellar erosions with calcification
within or a above sella.
- Most commonly seen in children but also present (+) in
adulthood
- Also ass with suprasellar calcification and polyuria
- Causes growth retardation because of hypothalamic pituitary
dysfunction.
-Rx is subfrontal or transspenoidal excision with adjuvant RT if
total removal not possible.
109. (2)

At least 50% of children with opsoclonus tonus-myoclonus


have an underlyiqgneuroblastoma.
Opsoclosus is a disorder of eye movement characterized by
involvements chaotic saccades that occur in all irections of
gaze, frequently associated with myodonus and ataxia.
Cause - 1. Idiopathic
2. Cancer related (paraneoplastic
e.g. Neuroblastomas in children and lung and breast CA in
adults.
110. (3)
Prolactinomas - 52%
GH cell adenomas 27%
Hormone Negative adenomas - 21%
Corticotropinomas - 20%
Thyrotropinomas 0.3%
111. (3)
"Intraductal papilloma is the most common of bloody nipple
discharge."
"Bloody discharge is more suggestive of cancer but is usually
caused by a benign papilloma in the duct"
CSDT
Also remember
- Nipple discharge is suggestive of cancer if it is spontaneous,
unilateral, localized to a single duct, occurs in women age of 40
years or more, is bloody, or is associated with a mass.
- Nipple discharge is suggestive of a benign condition if it is
bilateral or multiductal in origin, occurs in
Women age 39 years or less, is milky or blue green in colour.
112. (3)
An open biopsy is the most definitive investigation
- "The most reliable diagnostic test for breast cancer is open
excisional biopsy. When the sample is properly taken by the
surgeon and examined by the pathologist, such a test should
give no falsenegative and no false positive results."
- "Large-needle (core needle biopsy) is an accepted dlaqnostic
technique n which a core of tissue is removed with. a large
cutting needle. As in the case of nay needle biopsy, the main
problem is sampling error due to improper positioning of the
needle, giving rise to a false negative test result."
113. (3)
Macro calcification :
Macroclacification on mammogram indicates benign lesion

Opacity

Calcification

Breast
parenchyma
Nipple/areola
Skin
Coocer
liaaments
Ducts

Benign
Smooth margin
Low density
Homogenous
Thin halo
Macro calcification
(> 5 mmin
diameter)

Malignant
III defined margin, Irregular
stellate, speculated margin,
comet tail
High density
In homogenous Wide halo
Microcalcification (<5mm in
diameter)
Architectural distortion

Retracted
Normal
Normal
Normal

Retracted
Thickened
Thickened, increased
number
Focal dilatation

2014 NBE ONELINER


Subcutaneous
retro mammary
space

Normal

Obliterated

Calcifications are the most common finding on mammogram.


Most of these are benign calcification. The calcification
patterns which are associated with high probability of
malignancy are:
1. Clustered pleomorphic heterogenous macrocalcifications
There se small, usually less than 0.5 mm in diameter. vary in
size, shape and density.
Often termed granular calcification.
2. Fine linear or fine branching calcifications.
These calcifications are often seen in comedocarcinoma .
Architectural distortion
The structures of the breast are normally directed towards the
nipple. Disturbances in this normal orientation of tissue
towards the nipple, especially the tethering or pulling in of
structures to a point away from the nipple, is termed
architectural distortion.
Architectural distortion is highly suspicious for breast
carcinoma, although benign conditions such as radial scar,
surgical scar, fat necrosis can have this appearance.
114. (4)
According to latest AJCC classification the TNM stage would
be :
-T3N3,Mo
- Hence stage III
115. (2)
LCIS (lobular Ca in situ) I occurs bilaterally in 50 to 70% of
cases, while DCIS occurs bilaterally in 10 to 20%
of cases.
"Invasive lobular Ca is frequently multifocal, multicentric
bilateral.
116. (4)
Increased incidence with prolonged breast feeding
Paget's disease of the breast is a superficial manifestation of
an underlying breast carcinoma.
The underlying breast carcinoma is Intraductal carcinoma.
As already described, prolonged breast feeding has some
protective role in breast Ca.
Paget's disease of nipple :
- It presents clinically as a chronic eczematous eruption of
nipple, which may be subtle but may progress to an ulcerated
weeping lesion.
- It is usually associated with an DCIS on invasive ductal
carcinoma. The carcinoma mayor may not be palpable.
- Thus paget's disease is a superficial manifestation of the
underlying ductal carcinoma.
- Pathogomic histological feature is present of paget's cells in
the epithelium. Paget's cells are large pale vacuolated cells.
Treatment is described ahead.
117. (2)
Blockage of subdermal lymphatics:
Peaud's orange is produced due to obstruction of superficial
lymph vessels by cancers cells.
This causes edema of the skin giving rise to an appearance
like that of the skin of an orange (Peaud's orange appearance)

118. (3)
Simple Mastectomy
Since this patient is showing diffuse microcalcification on
mammography, a simple mastectomy would be the most
appropriate procedure.
Treatment of Ductal Ca in situ (DCIS)
DCIS carries a high risk for progression to an invasive cancer.
Schwartz writes - "The risk of invasive breast cancer is
increased nearly five fold in women with DCIS that was
originally detected, suggesting that DCIS is and anatomic
precursor of invasive ductal carcinoma."
The treatment of intraductal lesion is controversial. It can be
tit by either
- Simple mastectomy
- Breast conservation with wide excision (Lumpectomy) with
radiation therapy.
Choice of operation depends on both surgeon and patient
preference, however, following points must be kept in mind.
Simple mastectomy is the gold standard procedure with no
need of radiation and less recurrence rate (~2%), but there is
loss of breast, a big psychological trauma to the patient.
Breast conserving procedure is better cosmetically but
recurrence rate is more.
The patient also has to be monitored regularly both clinically
and by mammography for any recurrent disease.
Those in favour of conservative procedure, however say that
local recurrences of occur, can be successfully managed with
salvage mastectomy.
Simple mastectomy is recommended for the following DCIS
DCIS with evidence of widespread disease (involving two or
more quadrants) Mammographically identified multicentric
disease or diffuse suspicious calciflcation. Persistent positive
margins after re-excision.
Unacceptable cosmesis to obtain negative margins.
A patient not motivated to preserve her breast
Size greater than 40 mm
Comedo appearance on histology Negative estrogen receptor
status
High grade tumor
For patients with contraindications to radiation therapy
a) Prior radiation to the breast region
b) Presence of collagen vascular disease (SLE, Sclerodernia)
c) First or second trimester pregnancy
Axillary lymph node dissection is not necessary for DCIS.
Adjuvant Tamoxifen therapy is given to all DCIS patients.
Treatment of lobular carcinoma in situ (LCIS)
- LCIS is not an anatomical precursor of invasive disease like
PCIS rater it is considered as a risk factor for invasive breast
carcinoma (Which canb be either invasive or lobular type).
- The risk for invasive cancer is equal for both breast, so there
no benefit to excise LCIS.
- Treatment of LCIS is
- Observation with or without tamoxifen
- The goal of treatment is to detect the invasive cancer at an
early stage which will develop in about 25 to 35% of these
women.
- For patients who are unwilling to accept the increased risk of
breast cancer may be offered bilateral simple mastectomy.
Also remember
- The majority of invasive cancer which subsequently develops
in LCIS are of ductal nature (~65%) not lobular.

2014 NBE ONELINER


- One more difference between LCIS and DCIS.
119. (1)
All the standard hospitals in the world recommend breast
conservative tit for stage I and stage II cancers, but breast
conservativesurge. Surgery is an underutilized process. Still
mastectomy remains the more common treatment for breast
cancer and mastectomy radical mastectomy is the most
commonly performed procedure for Breast cancer.
- In stage IU and II
- Go for breast conservation surgery as the answer.
- Stage III
- Most of the stage III carcinomas can be managed by palliative
therapy. Curative therapy can be offered only in some cases.
- Stage IV
- Palliation is all that is possible in stage IV carcinoma.
Multicentricity means occurrence of a second breast cancer
outside the breast quadrant of the primary cancer (cf :
Multifocality refers to the occurrence of a second cancer within
vicinity or in the same breast quadrant as the primary cancer).
T4b Breast tumor implies - tumor of any size with skin
involvement (cf : T4a implies tumors of any size with chest
involvement) Both T 4a and T 4b are relative contraindication
for Breast Conservative Surgery.
So we are left with two options T1 breast tumor and
Extensive in situ cancer
T1 breast tumor (means tumor s 2 cm in size)
BCT is done in T1 tumors (unless some contraindications are
present)
Extensive is situ cancer
- Here the examiner is talking about extensive intraductal
component - EIC
- EIC is pattern of DCIS that is seen associated with invasive
carcinoma.
- The above mentioned book writes - 'At our institution EIC
positive tumors are defined as invasive carcinomas that show
the simultaneous presence of prominent DCIS within the
tumor (usually comprising 25% or more of the area of the
tumor) and DCIS beyond the edges of the invasive tumor. Also
included among EIC positive cases are tumors that are
predominantly DCIS with one or more foci or microinvasion'.
The significance of EIC positive tumors is that
In these tumors the intraductal involvement is more extensive.
than can be appreciated clinically or at the time surgery.
Therefore EIC positive patients who undergo a limited
resection of the clinically evident tumor frequently have
considerable residual DCIS in the vicinity of the tumor site.
This earlier led to increased recurrence in EIC positive tumors
managed by BCT.
But, now these tumors are not a contraindication for BCT, if
microscopically negative margins can be obtained. More
careful mammographic and pathological evaluation is
necessary on EIC positive tumors to define the extent of the
lesion and to determine the adequacy of excision.
But if one answer is to be chosen, it would be option (a) that is
T1 breast tumor .
120. (1)
Treatment of cystosarcoma phyllodes
a) TIT of small phyllodes tumour It should be locally excised
with an obvious 1 cm of normal breast tissue. After excision,
when the diagnosis of a phyllodes tumor with suspicious

malignant elements is made, re-excision of the biopsy site to


ensure complete excision of a tumour with a 1 cm margin of
normal appearing breast-tissue is indicated.
b) T/T of large phyllodes tumour Simple mastectomy.
Remember, cystosarcoma phyllodes do not metastasize by
lymphatic route, so axillary dissection is not needed.
Cystosarcoma phyllodes metastasize exclusively by blood
borne route
Other question on cystosarcoma phyllodes.
- Pathologically it is similar to fibroadenoma that is,
proliferation of both duct and stroma is seen.
- The tumor displays a range of behaviour from benign like
fibroadenoma to frankly malignant
- Peak incidence is in the fifth decade, exceedingly rare in
adolescents.
- The excised specimen is firm and rubbery and may separate
into leaf like structures therefore the term (phyllodes means
leaf Iike)
They usually present as a large growth which is nobile and
commonly there is a history of rapid growth.
121. (3)
122. (2)
(a) Cystisarcoma phyllodes : .
This tumor occurs predominantly in premenopausal women
around 40 years of age. However these may occur in
adolescents too. It presents as rubbery mass and on inspection
it shows big swelling with tense glistening skin over it.
(b) Fibrocystic breast disease:
Fibrocystic disease is also rare in adolescent. It is usually seen
in women of 20-40 years of age. It can be easily ruled out as it
presents with multiple masses associated with pain and
tenderness.
(c)Early Carcinoma
Carcinoma is rare in adolescents. Carcinoma can be easily ruled
out as it presents with hard lump.
(d) Virginal hypertrophy:
Although it often presents as bilateral breast enlargement, yet
the typical history of rapidly developing breast enlargement
and similar consistency of both the breast makes the diagnosis
of virginal hypertrophy likely. Virginal hypertrophy is a
massive sensitivity of the breast to oestrogenic hormones.
Treatment: Reduction Mammoplasty
Frequency of Breast masses in adolescents
- Fibroadenma.- 54%
- Virginal hypertrophy - 13%
- Fibrocystic or proliferative breast lesson - 24%
- Cystosarcoma phyllodes - 2%
123. (4)
Causes of Massive Enlargement of the Breast
1. Benign hypertrophy (usually bilateral)
2. Brodie's disease (Cystosarcoma phylloides)
3. Giant fibroadenoma
4. Sarcoma
5. Colloid Carcinoma
6. Filarial elephantiasis
124. (4)
Approximately 2 percent of American women who develop
carcinoma of the breast will be pregnant at the time of

2014 NBE ONELINER


diagnosis. The therapeutic approach to these patients has
changed considerably in recent years. Though changes in the
breast that occur during pregnancy often lead to a delay in
diagnosis of breast carcinoma, there is no convincing evidence
that breast carcinoma in pregnant women behaves differently
or its histologically different from that in non-pregnant
women. Furthermore, when patients are matched for age and
stage of disease, no significant differences in survival rates are
found. The majority of breast cancers in these patients as with
most premenopausal patients, is estrogen receptor negative
and not hormonally sensitive. Therefore elective termination
of pregnancy is generally longer indicated to decrease estrogen
stimulation of the tumor. Since radiation exposure endangers
the fetus and as there is no evidence that general anesthesia
and nonabdominal surgery increases premature labor,
modified radical mastectomy is recommended for stage I or II
carcinoma (tumor less than 4 cm in diameter). Patients in later
stages of pregnancy, however, can start radiation therapy
shortly after delivery, and some of them may be candidates for
breast conserving surgery and adjuvant radiotheraphy.
Chemotherapy does not appear to increase the risk of
congenital malformation when given in the second or third
trimester of pregnancy. Patients who require adjuvant
chemotherapy during the first trimester may opt for a
therapeutic abortion, however, since there is a slightly
increased risk of fetal malformation in that circumstance.
125. (2)
Paget's disease of the breast is unrelated to Paget's bone
disease. It represents a small percentage (1 to 3 percent) of all
breast cancers and is through to originate in the retroareolar
lactiferous ducts. It progresses toward the nippleareola
complex in most patients, Where it causes the typical finding of
nipple eczema and erosion. Up to 20 percent of patients with
Paget's disease have an associated breast mass, and these
patients are more likely to have involvement of axillary nodes.
Nipple areolar disease alone usually represents in situ cancer;
these patients have a 10 year survival rate of over 80 percent.
In contrast, if Paget's disease presents with amass, the mass is
likely to be an infiltrating ductal carcinoma. The generally
recommended surgical procedure for Paget's disease is
currently a modified radical mastectomy. The validity of breast
saving surgery and adjuvant radiation therapy for patients
without an associated mass is under investigations.
126. (1)
The most common are sodium disturbances, which
carfocurboth spontaneously and as a consequence of diuretic
therapy to control raised intracranial pressure. Both hypo and
hypernatremia can further compromise consciousness and
injudicious rapid correction can induce pontine myelinolysis.
127. (2)
Burst temporal lobe is the term sometimes used. to describe
the appearance of confusional intracerebral haematomas,
bleeding out into the subdural space from a disrupted cortical
surface. The blood follows the subdural space over the
convexity of the brain and appears as a concave hyperdense
collection.
128. (3)

Epidural hematomas are typically caused by a tear of the


middle meningeal artery or vein, or a dural venous sinus.
Ninety percent of epidural hematomas are associated with
linear skull fractures, usually in the temporal region. Only 2
percent of patients admitted with craniocerebral trauma suffer
epidural hematomas. The lesion appears as a hyperdense
biconvex mass between the skull and brain on CT scan. Clinical
presentation is highly variable and outcome largely depends
on promptness of diagnosis and surgical evacuation. The
typical history is one of head trauma followed by a momentary
alteration in consciousness and then a lucid interval lasting for
up to a few hours. This is followed by a loss of consciousness,
dilatation of the brainstem and death. Treatment consists of
temporal craniectomy.evaluation of the hemorrhage, and
control of the bleeding vessel. The mortality of epidural
hematoma is approximately 50 percent.
129. (4)
CSDT : "Posttraumatic epilepsy is more likely to develop if the
duramater was penetrated and generally manifests within 2
years following the injury."
Harrison : The superficial cortical scars that evolve from
contusions are highly epileptogenic and may later manifests as
seizures, even after many years."
130. (1)
"Extradural haemorrhage is an accumulation of blood between
the skull and the dura. In 90% of cases, bleeding is from a
branch of the middle meningeal artery".
Also remember
Subdural haemorrhage, commonest source is
communicating veins b/w the cerebral cortex and superior
saggital sinus.
Intracerebral haemorrhage, commonest source is
lenticulostriate arteries in the region around the basal ganglia*
and intemal capsule. It is frequently d/t rupture of CharcotBouchard aneurysms in the lenticulostriate arteries.
Subarachnoid heamorrhage - MC cause is rupture of Berry
aneurysms.

131.(2)
Chronic subdural haematomas. These haematomas are
most common in infants and in adults over 60 years of
age. They present with progressive neurological deficits
more than 3 weeks after the trauma. Often, the initial
head injury has been completely forgotten and the
pathology has been attributed to either dementia or a
brain tumour until patients are scanned. The initial
haemorrhage may be relatively small or may occur in
elderly patients with large ventricles or a dilated
subarachnoid space. Membranes derived from the dura
and arachnoid mater encapsulate the haematoma, which
remains clotted for 2-3 weeks then liquefies. The acute
clotted blood initially appears white on a CT scan. As it
liquefies, it slowly becomes black. Thus, there is a point
in time where it appears iso-dense with brain and all that
can be seen is apparent inexplicable shift on an otherwise
normal CT. These collections can then either resolve or
increase in size from osmotic effects or repeated small

2014 NBE ONELINER


bleeds. They are evacuated by drilling burr holes over the
collection and washing it out with warmed saline.
132. (3)
Steroids help to reduce peritumoral oedema in order to
reduce mass effect. Surgery is appropriate if there is a
solitary surgically accessible lesion and no systemic
spread, particularly if the primary site is unknown and
the histological diagnosis in doubt. Radiotherapy is used
to treat multiple metastases and following resection.
Stereotactic radiosurgery has been used in the treatment
of metastases smaller than 2 cm. with results similar to
those of surgery.
133. (2)
The treatment of choice for medulloblastoma is surgical
debulking and radiation therapy, which results in a 5
year survival of 56%. Chemotherapy with carmustine is
sometimes used in relapse, but is ineffective as an initial
treatment modality. Medulloblastomas are the most
common brain tumors in children. They are associated
with the roof of the fourth ventricle, not the floor.
Subarachnoid spread is seen in 35% of cases at time of
diagnosis.
134. (3)
Meningiomas are slow growing tumors that often present
with comR!;lintlif9t chronic headache. The olfactory
groove is a common location for meningiomas
135. (2)
Patient with Oligodendrogliomas classically present with
a long history of seizures. These tumors are often
calcified on computed tomography (CT) scan and most
commonly occur in the frontal lobe.

136. (3)
The finding of bilateral vestibular schwannomas
involving cranial nerve VIII is considered diagnostic for
neurofibromatosis type II.
137. (2)
Mannitol an osmotic dehydrating agent, Works by
drawing water from parts of the brain with an intact
blood-brain barrier. If this is disrupted, as in a cerebral
contusion, mannitol can leak out into the brain and
potentiate the mass effect. In head injuries it should
therefore, only be administered after consultation with a
neurosurgeon. It becomes ineffective when brain
osmolality becomes iso-osmolar with that of the serum.
Steroids work by stabilizing the blood-brain barrier and
reducing oxygen radical injury. Barbiturates give rise to
vasoconstriction. Frusemide reduces ICP by
reducing cerebral oedema and CSF production.

138. (3)
Lesch nodules (pigmented hamartomas of the iris) and
optic gliomas are two of the criteria that can be used to
diagnose neurofibromatosis type I. Other characteristics
include plexiform neurofibromas and autosomal
dominant inheritance.
139. (1)
These findings are suggestive of hypercortisolism. This
may be secondary to a number of sources including
adrenal tumors, exogenous steroid use, pituitary tumors,
and malignancies at other sites (e.g. small cell carcinoma
of the lung). Among the choices, Cushing's disease
defined as hypercortisolism secondary to an
adrenocorticotropic hormone (ACTH)-producing
pituitary adenoma is the most likely diagnosis.
140. (4)
A suprasellar calcification is typical of
Craniopharyngioma
Polyuria is d/t diabetes insipidus caused d/t
compression by the tumor over pituitary.
- "Clinically, patients with suprasellar tumors typically
present with diabetes insipidus, endocrinopathy" .
141. (4)
Dandy walker malformation is a cystic expansion.of 4th
ventricle in the post fossa due to failure of roof of 4th
ventricle development.
- 90% patients have hydrocephalus.
- As anomalies include agenesis of post. cerebellar vermis
and corpus callosum.
- Most children have evidence of long tract signs/
cerebellar ataxia & delayed motor and cognitive mile
stone.
- Management is shunting the cystic cavity in case of
hydrocephalus.
142. (2)
Berry aneurysm is me type of intracranial aneurysm
- Congenital weakness of wall
- Increased risk with Ehler Dan los /NF1, marfan's /
APKD /fibromuscular dysplasia.
- Predisposing factor - HTN and smoking.
- Rupture usually occurs at apex of sac leading to SAH or
Intraparenchymal hrg or both
- 90% occurs in Ant circulation.
- The wall is made up of thickened hyaline intima.
Rx - Embolisation /clipping of the Sac.
143. (3)
Fronal lobe lesion tends to present with personality
change, gait ataxia and urinary incontinence, Pathological
joking (Witzel Schultz syndrome). Parietal lobe lesions
are associated with sensory inattention, dressing apraxia,
asterognosis [a graphia, Lt-to-Rt Disorientation and

2014 NBE ONELINER


finger agnosia (Gertzman's Syndrome) if and dominant
side affected)]. Temporal lobe lesions cause hemiparesis,
contralateral superior quadratatopia and memory
disturbances; dysphagia. Occipital lesion associated with
incomplete contralateral Homonymous hemianopia.
144. (3)
Secondary haemorrhage occurs after 7-14 days & is due
to infection & sloughing of part of the wall of an artery.
(1), (2) & (4) are seen with reactionary
145. (3)
When oliguria occurs postoperatively, it is important to
differentiate between low output caused by the
physiologic response to intravascular hypovolemia and
the caused by acute tubular necrosis. The fractional
excretion of sodium (FE) is an especially useful test to aid
in this differentiation. Values of FE <1 percent in an
oliguric setting indicate aggressive sodium reclamation
in the tubules; values above this suggest tubular injury.
The fractional excretion is a simple calculation: (urine Na
X serum creatinine) + (serum sodium X urinary
creatinine). In the setting of postoperative hypovolemia,
all findings would reflect the kidney efforts to retain
volume: the urine sodium would be below 20 meq/L, the
urine chloride would not be helpful except in the
metabolically alkalotic patient, the serum osmolality
would be over 500 mOsm/kg, and the urine /serum
creatinine ratio would be above 40.
146. (4)
The risk of V. fibrillation increases at temp below 28C
and Cerebral metabolism is decreased by 6-7% of per
1drop in temp. Cardiac arrest occurs at 20C. The
Patients should be rewarmed for 24 hrs at room temp of
25-33C severe Hypothermia (< 28C) should be treated
as life threatening emergency. At temp < 30C, heart is
unresponsive to defibrillation and ionotropes so
medicines are best withheld until rewarming have been
achieved.
147. (4)
Half life of platelets is 9 days
RBC's adult = 120 days
V and VIII = 6-12 hrs.
Blood component
Temp for
Whole blood
1 - 6C
Packed cells
Platelets
20-24 C
Leucocyte conc.
20 - 24C
FFP
< - 30C
Thawed plasma
1 - 6 C
Albumin
Room Temp

Shelf life
35 days
42 days
5 days
24 hrs
1 yr
24 hrs
3 yrs

Cryoprecipitate
1 yr
< - 30C
- Frozen
20 - 24C
4 hrs
- Thawed
Stored blood lacks functional platelets after 24 hrs.
148. (3)
Symptoms of hemolytic transfusion in
Conscious patient
- Sensation of heat and oain in the
limbs
- Headache

Unconscious patient
- Diffuse Bleedina and
severe.
Hypotension
- Tachycardia

- Precardial and lumbar pain


- Tachv cardia and suffused face
- Oliguria and Hemoalobinuria

- Urticarial rash
- Raising airway pressure
on IPPV

149. (3)
Shock is defined as in adequate perfusion to maintain
normal organ function. Thus, the goal in the Rx of shock
is restoration is of adequate organ perfusion and tissue
Oxygenation.
Urine output is quantitative and relatively reliable
indicator of organ perfusion Adequacy of resucitation can
be determined by urine output and blood pH.
150. (2)
Platelet concentrate is required for patients with
thrombocytopenia.
FFP can be given in any of the congenital clotting factor
deficiency diseases, especially Christmas disease (factor
IX deficiency) or haemophilia B. Cryoprecipitate is a very
rich source of factor VIII.
151. (4)
The following are causes of retroperitoneal fibrosis also
known as retroperitoneal fasciitis or chronic
retroperitoneal fibroplasias.
1) Hodgkin's disease
2) Ca breast
3) Ca colon
4) Methysergide used for migraine headaches
5) Membranous glomerular disease
6) Inflammatory bowel disease
7) Leaking aortic aneurysm
152. (4)
There is a narrowing where the ureter is coursing in the
bladder wall & not before entering the bladder wall.
153. (3)
Stretching of renal capsule gives pain in the loin which
deep seated sickening ache.
Ureteric colic gives pain which radiates from loin groin,
the more distal the stone, the more the pain radiates to
the groin.
Severe inflammation of bladder causes wrenching
discomfort at end of micturation. Urethral pain is

2014 NBE ONELINER


scalding or burning during voiding felt in the vulva or
penis.
154. (1)
A stone in the ureter nearly aIways has its birth in the
kidney.
When the stone is in the intramural ureter the pain is
referred to the tip of the penis.
Urine is not always infected. If infection is present, then it
is an indication for surgical removal of the calculus.
Otherwise, expectant treatment is appropriate for small
stones that are likely to pass naturally.
155. (4)
It is a well known fact that cyanide is produced from
heated uric acid.
In Holmium YAG laser lithotripsy occurs, primarily
through a photo thermal mechanism that causes stone
Vaporization.
So production of cyanide when Holmium: YAG laser is
used for treatment of uric acid stones is considered a
potential side effect of this therapy. This has been
reported in vitro
This event has not been observed to produce any
adverse event in treated patients. This is probably due to
the fact that ablation site is continuously flushed with
liquid during the procedure.
More on Holmium YAG laser.
- Wavelength used is 2100 nm.
- Currently the HO : YAG laser is the most effective and
versatile Intracorporeal Lithotriptor with good margin of
safety.
- HO-YAG Laser Lithotripsy occurs primarily through a
photothermal mechanism that causes stone vaporization.
- They have the ability to fragment all types of stones
regardless of the composition of the stones.
- An important side effect of the laser is that it will melt
the wires of a basket or guide wire if it is fired directly
making extraction of the wire or the basket very difficult.
156. (4)
For staging the disease as, T1 or carcinoma in situ, or
even T2, histological examination is required as to the
invasion of lamina propria or muscle. Biopsy specimen
can only be obtained by cystoscopy; hence it is the right
answer.
157. (4)
"Ureteroscopic stone extraction is highly efficacious for
lower ureteral calculi" - Smith's Urology.
A ureteroscope is a long endoscope that can be passed
transurethrally across the bladder into the ureter. The
stones can be caught in baskets or endoscopic forceps.
Those that cannot be removed via baskets or endoscopic
forceps, are fragmented using ureteroscopic lithotrities.

Any renal calculi that is 4 mm or less in size, is allowed


some time (approx. 4 weeks) to pass spontaneously.
Indications for surgical removal of a ureteric calculus Stone is too large to pass spontaneousiy
- Urine is infected
- Repeated attacks of pain and the stone is not moving
- Stone is enlarging
- Stone is causing complete obstruction
- Stone is obstructing solitary kidney or there is bilateral
obstructions.
158. (2)
Risk factors for squamous cell ca of bladder
Schistosoma haematobium
- it's a risk factor for both TCC & sq. cell ca, but more for
sq. cell ca.
Chronic irritation from urinary calculi, long term
indwelling catheters, Chronic urinary infections
Bladder diverticula
159. (1)
Vesical caluli are common in some parts of the country,
e.g. Rajasthan, Andhra Pradesh and some north-eastern
states. These stones are usually single and composed of
ammonium and urate and calcium oxalate (ie
radioopaque stones). These calculi usually occur in boys
below 5 yrs. of age.
Most bladder stones contain calcium and are radiopaque,
cystoscopy is confirmatory because stones in the pelvis
seen on plain film may prove to be extravesical.
Treatment is usually transurethral Cystolitholapaxy."
Primary bladder stone is one that develops in sterile
urine. If often originates in a kidney and passes down the
ureter to the bladder, where it enlarges.
Secondary bladder stones occur in presence of some
infection, bladder outflow obstruction, impaired bladder
emptying or a foreign body such as nonabsorbable
sutures, metal staples or catheter fragments. Secondary
bladder stones are more common than primary.
Treatment of Vesical calculus :
Vesical calculus is removed usually by per-urtheral
litholapaxy.
Contraindication to per-urtheral litholapaxy.
A very large stone, a contracted bladder, a urethral
stricture that cannot be dilated sufficiently When the
patient is below 10 years of age.
Alternative procedures: Suprapubic lithotomy,
Percutaneous suprapubic litholapaxy.
160. (1)
Balkan nephropathy Transitional cell tumours of the
upper urinary tract have a very high incidence in certain
areas of the Balkans. The same population has a high
incidence of a form of primary nephropathy. The
causative agent has not been identified with certainty but
there seems to be an association with the consumption of

2014 NBE ONELINER


grain products stored in a damp environment. Tumours
that develop against a background of Balkan
nephropathy should be treated by nephron-sparing
surgery in view of the impaired overall renal function.
161. (3)
In case of extra peritoneal rupture of bladder a
cystogram shows extravasation of contrast material into
the pelvis around the base of bladder. The characteristic
'TEAR DROP' deformity represents bladder compression
by a large pelvic haematoma.
162. (2)
"Schistosomiasis - is the commonest cause of calcification
in the wall of bladder. Thin curvilinear calcification
outlines a bladder of normal size, and shape. Calcification
spreads proximally to involve the distal ureters
(appearing as two parallel lines) in 15%"
Schistosomiasis causes calcification in the wall of the
bladder which outlines the bladder. A full bladder will
appear as a fetal head in pelvis.
163. (2)
Urodynamic study is done to distinguish genuine stress
incontinence (due to sphincter weakness) from detrusor
instability in women and investigation of incontinence in
general.
164. (2)
In the first year of life, the bladder is closed following
osteotomy of both iliac bones just lateral to the sacroiliac
joints. Later, reconstruction of the bladder neck and
sphincters is required.
165. (1)
There is intermittent, painless, terminal haematuria seen.
Sandy patches on cystoscopy are the result of calcified
dead ova with degeneration of the overlying epithelium.
Considerable calcification of this nature is seen on
radiograph. Fibrosis is mainly the result of secondary
infection. The capacity of the bladder becomes much
reduced and contracture of the bladder neck may be
found. Carcinoma is a common end result in grossly
infected bilharziasis is of the bladder which has been
neglected for years. Urinary calculi, especially vesical and
ureteric, occur more frequently when bilharziallesions of
the bladder are present.
166. (2)
Ca bladder is common due to constant irritation of
bladder wall.
In males, the completely epispadiac penis is broader and
shorter than normal and bilateral inguinal herniae may
be present. There is uretero vesical efflux and
incontinence but no ventral curvature of penis.

167. (2)
The patient with incomplete bladder emptying & good
capacity may be managed by means of clean intermittent
catheterisation (CISC).
The patient with complete bladder emptying &
reasonable capacity with normal upper tracts may be
managed by means of condom drainage.
Patients with poor emptying, low capacity & upper tract
dilatation require addditional treatment.
168. (1)
The main complication is uretero ileal stricture which
can be limited by spatulation of the distal ureters & an
end-to- end anastomosis.
Stenosis at the ileocutaneous site is less frequent & a
short isoperistaltic conduit limits the formation of a
residual urine volume to reduce infection & avoid the
problems of reabsorption of urine.
169. (1)
The patient is suffering from genuine stress incontinence.
It is usually found in multiparous women with a history
of difficult labour often accompanied by the use of
forceps. It can be found in normal young women who
indulge in competitive trampooning & also in patients
with epispadias. The symptoms may change with the
menstrual cycle.
170. (2)
Cis-platinum plus gemcitabine given before
(Neoadjuvant) radical cystectomy has been shown to be
of benefit.
171. (4)
Full distension of the bladder is needed if searching for a
diverticulum. With inadequate distension of the bladder,
the mouth of the diverticulum is closed with epithelium
thrown into radiating pleats.
Ivu may give information regarding size of diverticulum.
(3) is done only during video urodynamic investigation &
will also give information about the emptying
characteristics of the bladder & diverticulum
172. (3)
The most common sites for superficial tumours are the
trigone & lateral walls of the bladder.
173. (4)
Oestrogen deficiency, which may give rise to lowered
local resistance seen in menopausal females predisposes
to UTI. Other causes are:
1. Meatal stenosis
2. Bladder diverticulum
3. Presence of calculus, foreign body or neoplasm.
4. Diabetes
5. Immunosuppression

2014 NBE ONELINER


174. (3)
Carcinoma is a common end result in a grossly infected
bilharziasis of the bladder which has been neglected for
years. It usually commences, not in a papilloma, but in an
ulcer & is therefore a squamous cell carcinoma (due to
metaplasia).
175. (4)
Frequency-dysuria syndrome or urethral syndrome is
more common in women & consists of (1), (2) & (3).
Carcinoma-in-situ, tuberculosis & interstitial cystitis
should be excluded.
General hygienic measures are applied as part of
treatment.
176. (4)
The physiological adhesions between the foreskin &
glans penis may persist until 6 years or more of age.
True phimosis consists of scarring of the prepuce, which
will not retract without fissuring. (1) Occurs later in life
in which the normally pliant foreskin becomes thickened
& will not retract.
177. (4)
Spontaneous remission is seen in 50% cases of the
disease. If remission does not occur, p-amino benzoic
acid powder or tablets or vitamin E tablets may be tried
for several months.
A number of operative procedures have been used after
excision of the plaque.
Placement of non absorbable sutures opposite the plaque
without its excision is Nesbitt's operation.
178. (2)
Giant condylomata acuminata is a precancerous
dermatologic lesion which is cauliflower like arising from
the prepuce or glans. It is difficult to distinguish from
well - differentiated squamous cell Ca.
Bowen disease is a squamous cell Ca-in-situ involving the
penile shaft. It appears as a red plaque with
encrustations. Erythroplasia of Queyrat or Paget's
disease of the penis, is a velvety, red lesion with
ulcerations that usually involve
the glans.
179. (4)
Patients who initially have clinically negative nodes but
in whom clinically palpable nodes later develop should
undergo a unilateral ilioinguinal node dissection.
Patients who have inoperable disease & bulky inguinal
metastases are treated with chemotherapy. In some
cases regional radiotherapy can provide significant
palliation.
180. (2)

Chronic lesions of Granuloma inguinale may become


grayish, especially the edges, where, after months or
years, malignant change may develop. The ulcerated area
bleeds' on touch & without treatment healing is only
partial & keloid is common.
Genital warts are sexually transmitted & may complicate
HIV lnfection.
181. (1)
By the age of 16 years, irreversible destructive changes
have occurred in the testis which halt spermatogenesis &
limit the production of androgens to around half of the
normal output.
182. (3)
This is a verrucous carcinoma affecting the penis. It is
locally destructive & invasive but appears not to spread
to lymph nodes to metastasise. Treatment is surgical
excision.
183. (4)
Priapism is idiopathic in 60% of cases, while the
remaining 40% of cases are associated with diseases (eg
leukemia, sickle cell disease, pelvic tumors, pelvic
infections, penile trauma, spinal cord trauma, or use of
medications. Currently, intracavernous injection therapy
for impotence may be the most common cause
184. (3)
Gonorrhoea is associated with infection of peri-urethral
glands and discharge per urethra.
Skin conditions like lichen planus & psoriasis affect the
penis. Drug hypersensitivity reactions can affect the skin
of the penis. Balanoposthitis is associated with penile
cancer, diabetes & phimosis. Monilia infections are also
quite common under the prepuce.
185. (1 )
Four chemotherapeutic agents demonstrate activity
against penile carcinoma. Bleomycin, methotrexate,
cisplatin & 5- FU.
186. (2)
Non filarial elephantiasis can result from fibrosis of the
lymphatics due to Lymphogranuloma venereum.
187. (2)
This is seen in middle aged and old men. Patients present
with complaints of painful erection, curvature of the
penis, and poor erection distal to the involved area.
The patient has no pain when the penis is in the nonerect state.
The condition has been noted in association with
Dupuytren's contracture of the tendons of the hand, in
which the fibrosis resembles that of Peyronie's disease
when examined microscopically.

2014 NBE ONELINER


Spontaneous remission occurs in about 50% of cases.
188. (2)
Patients with bulky retroperitoneal disease (> 3 cm
nodes or 3 or more 1- cm cuts on CT scan) or metastatic
NSGCT are treated with primary platinum-based
combination chemotherapy following orchiectomy. If
tumor markers normalize and a residual mass is
apparent on imaging studies, resection of that mass is
mandatory, because 20% of the time it will harbour
residual cancer, 40% of the time it will be teratoma and
40% of the time it will be fibrosis.
189. (1 )
Torsion of testis, which is the condition in question, can
take place spontaneously. Testicular torsion is most
common between 18 and 25 years of age and a few cases
occur in infancy. Elevation of the testis reduces the pain
of epididymo-orchitis and it makes it worse in torsion.
Arrangements should be made for early operative
fixation to avoid recurrent torsion if gentle manipulation
fails.
190. (1 )
By doing a transscrotal biopsy there will be metastatic
spread of testicular tumor along the needle track. Hence,
contraindicated. Scrotal U/S will assess primary
testicular tumor rapidly.
Inguinal exploration with cross clamping of the
spermatic card vasculature and delivery of the testis into
the field is the mainstay of exploration for a possible
testis tumor. If cancer cannot be excluded by
examination of the testis, radical orchiectomy is
warranted.
191. (3)
Circumcision soon after birth confers almost complete
immunity against carcinoma of the penis. Later
circumcision does not seem to have the same effect and
Moslems circumcised between the ages of 4 and 9 years
are still liable to the disease. (1), (2) & (4) are true
192. (1)
Leydig cell tumors are the most common non-germ cell
turners of the-testis & account for 1-3% of all testicular
tumors. Sertoli cell tumor comprises less than 1 % of all
testicular tumors. Gonadoblastoma comprise 0.5% of all
testicular tumors
193. (2)
AFP is not elevated at all in seminoma submit of HCG is
elevated in upto 7% of seminimas
194. (4)

Seminoma is the most common germ cell tumor in


bilateral testicular tumors, while malignant lymphoma is
the most common bilateral tumor of the testis
195. (3)
Partners should be warned that they should continue
with their contraceptive precautions until the success of
the operation has been confirmed by semen analysis
performed 12-16 weeks after surgery.
196. (2)
(1),(3) & (4) are false statements.
The seminal vesicle, which is affected, & the reservoir of
infection feel indurated & swollen on PR examination.
Lax hydrocele occurs in 30% of cases. Cold abscess is
formed in neglected cases over period of time.
197. (2)
The presence of contralateral atrophy or
ultrasonographic microlithiasis in patients with
testicular tumors warrants contralateral biopsy. If
diagnosed CIS is usually treated by external beam
radiation therapy.
198. (4)
For patients with bulky seminoma & any seminoma
associated with elevated AFP should receive primary
chemotherapy. Some of the successful regimens include
cisplatin, etoposide & bleomycin (PEB), vinblastine,
cyclophosphamide, Dactinomycin, bleomycin, & cisplatin
(VAB-6); & cisplatin & etoposide.
199. (2)
Metastasis to the testis is rare. The most common
primary site is the prostate, followed by the lung,
gastrointestinal tract, melanoma & kidney
200. (2)
The characteristics of a normal semen sample are:
Ejaculate volume - 1.5 - 5.5 ml
Sperm concentration >20 x 106 sperm I ml.
Motility> 50%
Forward progression: 2 (scale 1-4)
Morphology:> 30% WHO normal forms (> 4% kruger
normal forms)
201. (3)
The best treatment for gastric cancer is surgery. It's a
cancer so it has to be radical surgery. Since the tumour is
involving distal stomach and is considerably smaller we
have the liberty to spare some stomach. So we go for
subtotal radical gastrectomy. It would be accompanied
by D2 - Resection because node along named vessel
(celiac) is involved and needs to be removed.
202. (4)

2014 NBE ONELINER


The description in question is a classic presentation of
Acute Gastric Distension. It is commonly seen after
splenectomy and surgeries in region of fundus of
stomach. The management is usually conservative and
includes option 1, 2, & 3 surgery is not required. Due to
vomiting patient may have electrolyte disturbance which
need to be corrected accordingly.
203. (4)
Commoner variety of gastric volvulus is organoaxial. It is
usually associated with hiatus hernia, not Bochdalek
hernia. Investigation of choice is Barium swallow.
Endoscopy is infact dangerous, it may lead to perforation.
Transverse colon commonly herniates into the thoracic
cavity along with the stomach.
204. (4)
In this case we have involvement of liver and pancreas
along with Para aortic lymph node involvement all of
which indicate inoperability. The best palliation for
carcinoma stomach is again surgery. Radiotherapy has
poor role in carcinoma stomach. Palliative CT would be
the option if patient is not fit to undergo any surgery.
205. (3)
Most common benign tumour of stomach is epithelial
polyp. Adenomatous polyp is a premalignant condition
and is just one of the five types of gastric polyps
(Hyperplastic, Adenomatous. Hamartomatous,
Inflammatory and Heterotopic)
206. (4)
Treatment of primary gastric lymphoma is controversial.
Bailey says surgery should be the most appropriate
option and chemo reserved only for those with systemic
disease. The indication for surgery is bleeding,
perforation, obstruction, fistula formation and failure of
chemotherapy.
207. (3)
There has been a proximal migration in the site of
stomach cancer, but still all over the world about 40%
tumors occur in antrum, 30% in fundus and 25% at the
cardia. Tumours are more common on lesser curvature
than the greater curvature; Also Ca stomach is commoner
in lower socioeconomic group and in black males.
208. (4)
Hypertrophic pyloric stenosis causes metabolic alkalosis
with paradoxical aciduria. It is commoner in first born
male child. Ramsted's pyloromyotomy is the surgical
procedure. Vomiting is non-bilious.
209. (1)
Early postcibal (dumping) syndrome occurs within an
hour (usually 30min) of meals. It is due to rapid gastric

emptying of hyperosmolar contents into the small bowel


which then evokes a sympathetic response. It will be
aggravated by more food. Patient has got raised
hematocrit due to fluid shift.
210. (4)
Best treatment for bleeding is to stop the bleeding which
in case of duodenal ulcer is by ligation of the bleeding
vessel, which is most commonly the gastroduodenal
artery. Truncal vagotomy and andpyloroplasty is
treatment for recurrent ulcers.
211. (1)
Commonest cause of duodenal blow out leading to
duodenal fistula is post gastrectomy (Bilroth II). Other
causes are post operative after surgeries on biliary tract,
duodenum; pancreas, right colon, aorta and kidney. Only
15% are as a result of trauma, perforated ulcers and
cancers.
212. (2)
Upper GI bleed is due to either bleedipg peptic ulcer or
oesophageal varices. In an acute episode due to
esophageal varies endoscopic sclerotherapy ligation is
effective. If this fails then one tries balloon tamponade.
Vasopressin causes splanchnic vasoconstriction and
reduces bleeding in an acute episode. Blockers are used
for preventing the recurrence of bleeding. They are not
used in acute setting.
213. (3)
Diarrhea occurs in postgastrectomy patients. Calcium
deficiency, vitamin B12 deficiency and steatorrhoea occur
due to malabsorption.
214. (1)
Malignant change does not occur in duodenal ulcer. In
case of perforation patient will present with acute
abdomen. The symptomatology in question is typical of
gastric outlet obstruction which is most commonly
associated with long standing peptic ulcer disease and
gastric cancer. Endoscopic dilatation may be tried, but
operative surgery is usually required.
215. (3)
The vomiting of hydrochloric acid results in
hypochloremic alkalosis. Initially urine has a low chloride
and high bicarbonate content reflecting primary
metabolic abnormality. This bicarbonate is excreted
along with sodium and so with time the patient becomes
progressively hyponatremic and more profoundly
dehydrated. Because of dehydration a phase of sodium
retention follows and potassium and hydrogen ions are
excreted in preference. This results in urine being
paradoxically acidic and hypokalemia ensues.

2014 NBE ONELINER


216. (2)
The patient has presented with a recurrent disease. So
the main aim of surgery now should be to have least
recurrence. Amongst the given options, option (2) has
the least recurrence rate (only 1%), highly selective
vagotomy has least mortality and morbidity but maximal
recurrence rate (2-10%). Truncal vagotomy is not done
in isolation, it is always accompanied by a drainage
procedure.
217. (2)
Duodenoduodenostomy is the best option. Duodenal
atresia occurs at the point of fusion between foregut and
midgut and therefore lies in neighbourhood of ampul!a of
Vater. It occurs due to failure of complete recanalisation
of duodenum in utero. Treatment is
duodenoduodenostomy which is constructed using most
dependant portion of proximal atretic segment to
minimize stasis.
218. (1)
Jejunoileal bypass was the first surgical approach to
morbid obesity and was effective in reducing weight.
However the procedure is widely practiced because of
multitude of complications. Most serious of which is liver
disease. This was secondary to protein calorie
malnutrition and bacterial overgrowth in bypassed
segment. None of the patients operated for morbid
obesity can maintain weight reduction unless he follows
a strict dietary regimen. Ulceration after roux-en-y
gastric bypass does not occur in antrum, it occurs in the
small intestinal part anastomosed to stomach.
219.(2)
Secondary diverticuli are the ones which occur in 1st
part of duodenum and they are usually the results of
scarring following duodenal ulcer. They are false
diverticula. 90% of primarily diverticula are solitary and
80% are in second part of duodenum commonly on its
concavity, in region of Ampulla of Vater. Most of them are
asymptomatic and found incidentally.
220. (4)
Mesenteric arterial syndrome also known as superior
mesenteric artery syndrome results from compression of
third part of duodenum between superior mesenteric
artery and aorta. This condition occurs most commonly
in young asthenic individuals, with females more
commonly involved than males.
Predisposing factors include
Weight loss
Supine immobilization
scoliosis
placement of body cast (cast syndrome)
Symptoms include nausea, vomiting, abdominal
distension, weight loss, postPrandial epigastric pain.

Diagnosis is by Barium meal and hypotonic


duodenography.
Treatment initially conservative and is usually successful.
Operative treatment is Duodenojejunostomy
221. (1)
Laparotomy and tube duodenostomy makes it a
controlled fisula and feeding jejunostomy allows enteral
feeding. So option (1) is the best. Reanastomosis in this
scenario would lead to leak hence should not be
attempted. Enteral
feeding should always be preferred over parenteral
feeding.
222. (4)
The band extends from 2nd part of duodenum of the right
Paracolic gutter. This anatomical location of band would
most commonly obstruct duodenum.
223. (2)
Coil spring or stacked coin is seen on Barium Swallow in
case of duodenal hematoma. Although characteristic of
intramural duodenal hematoma. This finding is present
only in 25% of patients with duodenal hematoma.
Operative exploration and evacuation othematoma may
be considered after 2 weeks of conservative therapy.
Most of duodenal hematomas resolve by conservative
management consisting of nasogastric aspiration and
total parenteral nutrition.
224. (1)
Duodenal atresia can be classified into
Type I (most common) - mucosal web with normal
muscular wall (windsock deformity) Type II short fibrous
cord connecting two atretic ends
Type III (least common) - complete separation of atretic
ends
An intrinsic obstructive web is known as windsock
deformity
225. (2)
The most dangerous type of esophageal perforation
amongst given options is Boerhaave's syndrome, because
here the esophagus perforates under pressure leading to
massive mediastinal spillage of contents. Iatrogenic
perforation is the commonest type of esophageal
perforation.
226. (3)
Angiodysplasia is seen above 60 years of age. Ascending
colon is the commonest site.
Selective angiography is the best of investigaion and
cannstigation and can also be used to embolize the
bleeder.
Angiodysplasia is associated with arotic stenoses

2014 NBE ONELINER


227. (4)
Abdominal actinomycosis usually presents as a right iliac
fossa mass 3 - 4 weeks after an appendicectomy.
Penicillin is the drug of choice
228. (4)
Paralytic ileus is the commonest complication in typhoid,
which presents as a non-obstructive intestinal
obstruction. Haemorrage is the leading symptom.
A perforation usually occurs in the third week.
229. (4)
The fascial condensation which separates the rectum
from the prostate is the Denonvillier's fascia. The
Waldeyer's fascia separates the rectum from the sacrum
and coccyx.
230. (1)
5 Fluorouracil in combination with leucovorin in the
chemotherapy of choice in rectal cancer. Newer drugs
like irinotecan and oxaliplatin have shown good promise.

231. (2)
It is important to confirm the absence of a mechanical
cause of the obstruction by colonoscopy or single
contrast water soluble bariums enema. Colonoscopy is
also the treatment of choice for decompression. It may
recur in 25% of cases. If colonoscopy fails, a cecostomy
may be required.
232. (1)
Synchronous malignancy means two separate sites of the
colon having a malignancy or the same tumour within 6
months. This is seen in 5% of cases. Hence, it is important
to have a coloscopy for all patients of rectal or sigmoid
cancer in order to look for a synchronous cancer in the
caecum/transverse or ascending colon.
233. (2)
It is usually a clockwise twist of the caecum - obstruction
may be partial. Caecopexy or Caecostomy is the
treatment of choice.
234. (1)
Sigmoidoscopy done is a patient with acute diverticulitis
of colon shows inflammation of the mucosa.
The X-Ray abdomen in acute diverticulitis shows saw
tooth appearance of the mucosa rigid sigmoidoscope
could be passed upto 16 cm and flexible sigmoidoscope
upto 60 cm of colon.
235. (3)
Presence of Pseudomonas, Klebsiella and proteus is urine
of a child of high imperforate anus is suggestive of
communication between the anus and bladder.

236. (4)
Lower gastrointestinal bleeding is defined as a bleeding
from a site distal to the ligament of Treitz.
"Hemorrhoids and anal fissure are the most common
cause of lower G. I. bleeding however the bleeding is
rarely massive",
Angiodysplasia and diverticulitis are the other two
common causes of bleeding and they usually present
with massive bleeding:
Here are the causes of acute lower GI bleeding :Common
- Angiodysplasia
- Diverticulosis
- Anorectal disease
Less common
- Neoplasia Carcinoma, polyps
- Colitis Radiation, ischaemic, ulcerative
- Infective Enteric fever, amoebic ulcer, T.B.
HIV related (gonorrhoea, CMV.)
Rare
- Meckel's diverticulum
- Intussusception
237. (2)
Rectal polyps are most commonly seen in juvenile
patient. The most common presentation of rectal polyp is
painless bleeding per rectum. Meckel's diverticulum is
most commonly present as diverticulitis and this causes
pain in abdomen with bleeding P/r
238. (4)
Not only a solitary but even multiple metastasis is not a
contraindication for surgery because surgery is also the
best palliative measure.
If only a solitary liver metastasis (even upto 3 static
lesions) is present it can be later resected with good
prognosis. Carcinoma of the left side of the colon usually
of stenosing variety. The main symptoms are those of
increasing intestinal obstruction, altered bowel habits;
tenesmus and bleeding.
Anemia (severe and unyielding) is seen in right sided
lesions.
Mucinous and 'signet ring' cell varities have poor
prognosis.
Colorectal carcinomas are resistant to most
chemotherapeutic agents. Recently 5 FU and levamisole
have been found to be of some use. They are used as
adjuvant chemotherapy in Dukes stage C lesions.
239. (2)
Although diverticulosis involves mainly the left side of
colon, bleeding occurs mostly from right side of colon.
Right side is supplied by superior mesentric artery.

2014 NBE ONELINER


"Hemorrhage from colonic diverticula is the most
common cause of hematochezia in patients over the age
of 60."
240. (3)
"In solitary rectal ulcer syndrome, one or more ulcers are
present in the distal rectum, usually on the anterior
wall." Schwartz
Causes of solitary rectal ulcer syndrome
o Due to a combination of
- internal intussusception
- anterior rectal wall prolapse
- increase in intra rectal pressure
This combination of factors is usually due to chronic
straining as a result of constipation.
Symptoms pain, bleeding, mucus discharge or outlet
obstruction
Treatment
o Nonoperative therapy (high fibre diet, defecation
training to avoid straining, and laxatives or enemas) is
effective in the majority of patients.
o Surgery (either abdominal or perineal repair of
prolapse) is reserved for highly symptomatic patients,
who have failed all medical intervention.
241. (2)
Digital reposition must be taught to the parents. If it fails
after 6 weeks of trial, 5% phenol in almond oil is injected
under GA Only rarely is surgery required.
242. (2)
Most authors claim a 2 cm margin distally to be safe in a
Ca rectum. This forms the premise in doing low and ultra
low anterior resections with the advent of staples.
243. (4)
The internal sphincter is composed of longitudinal, non
striated involuntary muscles supplied by autonomic
nerves. It is a continuation of the rectal muscle coat.
244. (4)
It usually takes 6 hours for sufficient air to collect in the
large intestine to cast a radiological shadow. Only then
will the distance between the metal coin and air column
be measured and 'high' anal agenesis can be
distinguished from 'low' oral agenesis.
245. (3)
By far, the most frequent cause of a tubular inflammatory
stricture of the rectum is LGV. Frei's test is usually
positive.
80% of patients are women. Antibiotics are curative in
80%. 20% will require surgical excision.

The various levels which can be examined by various


methods are:
1) Digital examination
8 - 10 cm
2) Proctoscopy
8 - 12 cm
3) Rigid sigmoidoscope
18 cm
4) Flexible sigmoidoscope
60 cm
5) Colonoscope
160 cm
247. (1)
It can be caused by Spirochaeta vincenti and bacillus
fusiformis. Inflamed mucosa with pinpoint bleeding looks
like a strawberry. Treatment of choice is acetarsol
suppositories with vitamin C
248. (2)
Anorectal ring marks the junction between rectum and
anal canal.
It composes of the puborectalis, deep external sphincter,
longitudinal muscle and highest part of intemal
sphincter.
249. (1)
Commonest site for an anal fissure (90%) is middle
posterior next most common is the anterior midline.
250. (4)
In anal canal cancers, presently concurrent chemo
radiotherapy is the treatment of choice. Usually a
combination of 5-FU and mitomycin is given.
If the lesion does not solve completely on CT & RT,
Abdominoperineal resection is done.

251. (2)
Patients with Insulinoma do not have weight loss.
Insulinomas are usually benign and solitary and hence
can be enucleated during surgery.
252. (3)
Areas of ductal dilatation altemating with areas of ductal
stenosis are common findings in alcoholic patients who
have severe chronic pancreatitis. This type of duct
obstruction cannot be relieved by sphincteroplasty
because of multiple areas of stenosis along the duct.
Although total pancreatectomy would be a beneficial
approach, morbidity and mortality with this procedure is
extremely high. Thus the procedure of choice in such
cases is side to side pancreaticojejunostomy
253. (4)
ERCP is rarely required in acute setting as management
is essentially conservative.
Serum amylase rise is nonspecific and occurs in injuries
of any intraabdominal organ.

246. (2)
254. (2)

2014 NBE ONELINER


Symptomatic annular pancreas has to be treated.
The treatment involves bypassing the obstruction.
Duodenoduodenostomy is the most physiological
jejunostomy.
255. (3)
Clinical signs and symptoms of intra abdominal
abscesses are often minimal and delusory. However,
recurring and persisting fever that initially is
intermittent and occurs in almost all patients, as
leucocytosis, shift to left. Paralytic ileus, abdominal
distention, anorexia and vomiting also may occur. With
subphrenic abscess pain in upper abdomen and lower
chest, associated with respiratory, difficulties may be
present. In this condition, plate like atelectasis or pleural
effusion can be seen on chest x ray above affected side of
diaphragm in 90% of cases, and an abnormal air fluid
level can be seen in abscesses 25% of the time. Although
pulmonary embolism and pancreatitis are possible
causes of similar clinical findings, they would not account
for abnormal air fluid level found in left upper quadrant
of
patient presented in the question.
256. (1)
Most common tumour of pancreas is adenocarcinoma.
Endocrine tumours are rare. Amongst them insulinoma is
most common.
257. (3)
Management of pancreatic abscess is to let the pus out by
external drainage combined with antibiotics. The
procedure is called as Beger's procedure.
258. (2)
Cholecystectomy
Mucocele of the Gall bladder - It is one of complication of Gall stones.
- Caused due to obstruction of the stone at the neck of the
bladder.
- In course of time the bile is absorbed and replaced by
the mucus secreted by the Gall bladder epithelium.
Due to this the Gall bladder may because distended and
palpable.
Treatment
- The tit is early cholecystectomy
- If early tit is not done following complication can occur
- Empyema.
- Perforation
- Gangrene}:
259. (2)
Gall stone pancreatitis
The prognosis is best in patient where pancreatitis is
caused by a remediable cause such as cholelithiasis.

'Eradication of gall stone disease' prevents further


attacks of pancreatitis.
260. (2)
Ileal resection or ileal disease decreases enterohepatic
circulation of bile salts (or acids). This decreases the
biliary secretion of bile salts, thus increasing the
cholesterol bile acid ratio.
261. (4)
Acalculous cholecystitis does not predispose to
carcinoma gall bladder. Also oral contraceptives are not
associated with carcinoma gall bladder. Porcelain gall
bladder, Cholecystoenteric fistula, adenoma, choledochal
cyst, gall stones, cholangitis and typhoid carriers are all
at risk for development of carcinoma gall bladder.
262. (3)
Triad for diagnosis of choledochal cyst is
Pain abdomen
Progressive Jaundice
Lump in abdomen
263. (1)
ERCP will help to diagnose the tear and will be helpful
in stent insertion.
Hepaticojejunostomy is done only if there is loss of
length of hepatic duct
Primary repair after exploration is a morbid procedure
as compared to ERCP and hence will not be done.
USG guided insertion of drain will also be required in
addition to ERCP and stenting, but is not the primary
treatment for CBO tear.
264. (4)
Cholecystectomy is not indicated for asymptomatic
patients with gall stones as a routine because less than
25% of patients with asymptomatic gall stones will
develop symptoms that require intervention. However
one may have to do cholecystectomy for asymptomatic
gall stones if stone> 3 cm, patient is a diabetic, multiple
small stones, Gall bladder with polyp or calcification.
265. (1)
Gall stones predispose to carcinoma gall bladder, not
cholangiocarcinoma. Other options are known
predisposing factors for cholangiocarcinoma, others are
schistosomiasis, Biliary enteric anastomosis and
Choledochal cyst.
266. (1)
This patient has a recurrent CBO stone. The treatment of
choice for recurrent / retained CBO stone is ERCP and
stone retrieval. But since the stone measures 2.5 cm it is
too large to be extracted via ampulla of Vater. Hence

2014 NBE ONELINER


treatment here would be supraduodenal
choledochotomy and CBO exploration.
267. (4)
Type I choledochal cyst is a fusiform dilatation and
excision is mandatory as is a premalignant condition.
Roux loop is required as end to end anastomosis is not
possible with bile duct.
268. (4)
Patients with symptomatic retained stones should
undergo endoscopic sphincterotomy after papillotomy
and retrieval of stones. Success rates between 85-95%
have been reported. It seems best suited for elderly, poor
surgical risk patients as it is least invasive. Patients in
whom endoscopic sphincterotomy and stone extraction
has been unsuccessful, extracorporeal shock wave
lithotripsy has proven beneficial.
269.(4)
Mucocele of gall bladder is nothing but sterile mucus
accumulation in gall bladder. Laparoscopy is
contraindicated in cirrhosis due to high risk of bleeding,
in prior upper absominal surgery due to adhesions, and
in suspected Ca gall bladder for fear of incomplete
excision of malignancy.
270. (3)
A T-tube cholangiogram is taken on the 7th to 10th postop day to ensure there are no retained stones. If the
cholangiogram is normal, the T-tube is clamped.
Removed for the T-tube will depend on the material from
which it is made, as this will determine the length of time
for a track to form. If it is latex rubber, the T-tube can be
removed at 10-14 days, but if it sialistic it should be left
for 3-4 weeks before removal. Following removed of the
T-tube there may be a small bile leak that persists for
1~2 days."
271. (3)
Splenic artery aneurysms are commoner in females
(2:1) hence option 3 is the answer.
Usually related to main arterial trunk
Mostly single and symptomless.
Occasionally may cause an audible bruit in left
hypochondrium and opacity in plain X ray abdomen due
to calcification in the sac.
They have to be operated as mortality is high if rupture
occurs. Procedure of choice is splenectomy and removal
of diseased artery.
272. (3)
Hereditary spherocytosis
"Splenectomy is the sale treatment for hereditary
spherocytosis and is indicated even when the anemia is
fully compensated and the patient is asymptomatic."

In thalassemia, since the anemia is due to both


increased destruction of red 'cells and decreased
hemoglobin production, splenectomy does not cure
anemia as in spherocytosis, but it may reduce transfusion
requirement.
Splenectomy is very rarely indicated in sickle cell
anemia (in 3% of pts) and the most. frequent indication
is hypersplenism and acute sequestration crisis.
Although there is no benefit of splenectomy on the
disease process.
273. (1)
The cause for recurrent symptoms is residual splenic
tissue, which are accessory spleens and are usually
missed during splenectomy. All the mentioned options
except option 1 are common sites for accessory spleen.
274. (4)
Insulinoma is equally distributed throughout the
pancreas. Classical feature of insulinoma is fasting
hypoglycemia relieved by intake of glucose.
GASTRINOMA is seen in the gastrinoma triangle i.e. the
Triangle of passero.
275. (1)
Transduodenal sphincteroplasty done via ERCP guided
catheter is treatment of choice for sphincter of odd
spasm/stricture or stenosis.

276. (3)
Lockwoods' repair is low approach for the repairs of a
femoral hernia.
Lotheissein's repair is an inguinal approach to repair of
femoral hernia through the incision over inguinal canal
as for inguinal hernia.
Stopa's repair is great prosthetic reinforcement of
peritoneal sac, which tackles both inguinal as well as
femoral hernia.
Moloneys darning is done only in repairing lnguinal
hernia.
277. (1)
In children the omentum is small and under developed.
Hence generalised peritonitis occurs early. Ingestion of
food, enema stimulates gut motility and hence hinders
localisation. Option (3) causes immunosuppression.
278. (4)
This is because contractions of the bladder commence at
apex and pass towards the base. Hence a patent urachus
which opens into the apex of bladder in temporarily
closed during micturition. This fistula will therefore
present only when there is obstruction to urinary
outflow.

2014 NBE ONELINER


279. (1)
Bochdalek hernia is posterolateral congenital
diaphragmatic hernia. Prenatal ultrasound shows
herniated gut in thoracic cavity. It usually occurs on left
side and hence stomach and transverse colon are most
common contents.

tubercle. It contains perivesical fat and portion of


bladder. Since it occurs through a defect in abdominal
wall it is a type of direct hernia.

280. (4)
Conservative treatment is successful in 93% of cases
hence it is preferable to wait for 2 years. This is a case of
congenital umbilical hernia. Coin strapping is advised
upto 2 years. If hernia persists beyond 2 years,
herniorrhaphy is indicated.

289. (4)
Progressive bacterial synergistic gangrene is due to
synergism between bacteria from the intra peritoneal
environrnent and those colonizing the skin. The
maximum intraperitoneal contamination among given
option is with option 1 and 4. But in option 1 surgery
done is colostomy which affectively drains out all the
intraperitoneal contamination. So the answer would be
option 4.

281. (3)
Desmoid is a fibroma, which usually occurs in
musculoaponeurotic tissues of abdominal wall especially
below umbilicus. It can also occur in old hernia scar and
intraperitoneally.

290. (3)
Most common presentation of tuberculous paritonitis is
pain in abdomen (present in 90%) followed by fever
(60%), loss of weight (60%), ascites (60%)night sweats
and abdominal mass.

282. (1)
Although divarication of recti is seen principally in
elderly women and multipara the form that limits the
divarication to above umbilicus is commoner in babies.

291. (3)
Exomphalos also mean omphalocle or umbilical hernia. It
is a disease of abdominal wall because it results from
failure of all or part of gut to return to the coelom during
early fetal life. Exomphalos has covering of amnion and
peritoneum where as gastroschisis have no coverings.

283. (4)
Secondary carcinoma at umbilicus 'Sister Joseph's node'
is due to primary in stomach, ovary colon and breast. The
spread is believed to be along the falciform ligament
from liver mets.
284. (3)
Age female sex and relation to menses clinch the
diagnosis of Endometrioma. Raspberry tumour usually
presents in childhood.
285. (4)
Pregnancy and female sex are predisposing factors for
femoral hernia as they contribute to laxity of ligaments.
However femoral hernia is also seen in males. It is most
likely to strangulate.
286. (4)
Raspberry tumour is exuberant granulation tissue
pouting out from the unobliterated distal portion of
vitellointestinal duct.
287. (4)
Familial Mediterranean fever also known as periodic
peritonitis is commoner in females. Colchicine is not the
causative agent it is used in prevention of recurrent
attack of the disease.
288. (1)
Prevesical hernia is a narrow necked hernia through a
gap in medial. panpfonjoint tendon just above pubic

292. (3)
Serosanguinous discharge is the forerunner of disruption
in 50% cases and signifies that intraperitoneal contents
are lying extraperitoneally. Pain and shock are absent.
Signs of intestinal obstruction may be present, but is not
usual. An emergency operation is required to replace
bowel, relieve obstruction and resuture wound. There is
biochemical evidence that healing after disruption
produces stronger wound.
293. (2)
Colonic perforation during colonoscopy is usually
detected early (within 6h - of perforation) and also the
colon is already prepared before taking up patient for
colonoscopy. Hence primary closure of defect should be
done.
294. (1)
Omphalitis is infection of umbilical cord stump.
Staphylococcus aureus is the most common organism
responsible, followed by Streptococci, E. Coli and
Clostridium tetani.
295. (2)
Hematoma of rectus sheath is due to tearing of inferior
epigastric artery, which is common in elderly females,
muscular males, in multipara and in late pregnancy.
Distinguishing between rectus sheath hematoma and
strangulated Spigelian hernia is difficult. Absence of

2014 NBE ONELINER


vomiting suggests hematoma and presence of resonance
over swelling favours Spigelian hernia.
296. (2)
Colpotomy is drainage of pelvic abcess via vaginal
fornices.
297. (4)
Infection leads to poor wound healing and subsequent
weakness of the incision site obesity is also a leading
cause, but not the most common. Persistent post
operative cough and post operative abdominal distension
are risk factors.
298. (3)
Abdominal dehiscence after an abdominal surgery is
most likely to occur between 6th and 8th post operative
days.
299. (1)
Lytle's method involves narrowing of the deep inguinal
ring so that the peritoneum does not enter thru it into the
inguinal canal. Sac enters through deep inguinal ring in
indirect hernia.
300. (1)
Obturator hernia through the obturator foramen is six
times commoner in females. Most patients are over 60
years. It is usually a Ritcher type (containing a part of
circumference of bowel within the sac). It usually
presents as a swelling in femoral triangle but a PV/PR
examination may reveal a tender lump in relation to
obturator foramen.

Das könnte Ihnen auch gefallen